You are on page 1of 131

Applied Elasticity

Marijan Dravinski
September 2000

Applied Elasticity (ME 509)


by
M. Dravinski

Chapter 1

Tensors and Dyadics


1.1

Cartesian Tensors

If we have two sets of right-handed Cartesian coordinate systems {xi } with the
basis {ei } and {x0i } with the basis {ei }, i = 1, ..., 3, with the same origin, the
coordinates of a point P can be expressed in both coordinate systems (Fig.1.1
). Then,
x = x1 e1 + x2 e2 + x3 e3 = xi ei
x = x01 e01 + x02 e02 + x03 e03 = x0i e0i

(1.1)
(1.2)

where summation over repeated indices is understood. Thus, we have


uv
uv
uv

= (ui vi )ei
= ui vi

e1 e2 e3
= u1 u2 u3
v1 v2 v3

(1.3)
(1.4)
(1.5)

= (u2 v3 u3 v2 )e1 + (u3 v1 u1 v3 )e2 + (u1 v2 u2 v1 )e3

(1.6)

where
ei ej
ei ei
e2 e3
e3 e1
e1 e2

=
=
=
=
=

ij
0, i = 1, 2, 3
e3 e2 = e1
e1 e3 = e2
e2 e1 = e1

(1.7)
(1.8)
(1.9)
(1.10)
(1.11)

Then from Eqns. 1.1 and 1.2 it follows that


x01 = e01 e1 x1 + e01 e2 x2 + e01 e3 x3
2

(1.12)

X3

X3

X2
X2

X1

X1

Figure 1.1: Two Cartesian coordinate syatems.


or
x01 =

11 x1

12 x2

13 x3

(1.13)

with
= e01 e1 = cos ]{x01 , x1 }
= e01 e2 = cos ]{x01 , x2 }
= e01 e3 = cos ]{x01 , x3 }

11
12
13

(1.14)
(1.15)
(1.16)

Similarly, we have that


x02
x03
where
ij

=
=

2k xk

(1.17)
(1.18)

3k xk

= e0i ej = cos ]{x0i , xj }

(1.19)

Or in matrix notation
x0 = Lx;

L=[

ij ]; i, j

= 1, 2, 3

(1.20)

The last result can be written in index notation as


x0i =

ij xj ; i, j

= 1, 2, 3

(1.21)

Therefore, Eq. 1.20 represents transformation of vector x form coordinate system {xi } to coordinate system {x0i } with matrix L denoting direction cosine
matrix or linear transformation matrix.
It is evident from Eq. 1.20 that
e0i = Lei ; i = 1, 2, 3
3

(1.22a)

But since the basis {e0i } are orthonormal, i.e.,


0
e0i e0j = e0T
i ej = ij

it follows that

e0T
1

e01
LT L = e0T
2
e0T
3

e02

e03

(1.23)

=I

(1.24)

or that the transformation matrix L is an orthogonal matrix. Therefore, it


follows from Eq. 1.20 that
(1.25)
x = LT x0

1.1.1

Index Notation

Throughout we shall employ the following index notation conventions:


Range convention: Every index takes on values 1,2, and 3 unless stated
dierently,
Summation convention: Repeated (dummy) indices are summed up over
their range, and
Free index convention: Indices that are not dummy one are called free
indices.

1.1.2

Alternating Symbol

This symbol is defined as following

1 for even permutations of i,j,k


0
otherwise
=
ijk

1 for odd permutations of i,j,k

(1.26)

Consequently, we can write

ei ej
(ei ej ) ek
uv
(u v)i

=
=
=
=

ijk ek
ijk
ijk uj vk ei
ijk uj vk

(1.27)
(1.28)
(1.29)
(1.30)

Namely,
uv

= 1jk uj vk e1 + 2jk uj vk e2 + 3jk uj vk e3


= ( 123 u2 v3 + 132 )e1
+( 231 u3 v1 + 213 )e2
+( 312 u1 v2 + 321 )e3
= (u2 v3 u3 v2 )e1 + (u3 v1 u1 v3 )e2 + (u1 v2 u2 v1 )e3
4

(1.31)
(1.32)
(1.33)
(1.34)
(1.35)

Example 1 Show that

ijk ijk

= 6.

Solution 1
ijk ijk

=
=

+ ij2 ij2 + ij3 ij3


i21 i21 + i31 i31
+ i12 i12 + i32 i32
+ i13 i13 + i23 i23
= 321 321 + 231 231
+ 312 312 + 132 132
+ 213 213 + i23 i23 = 6
ij1 ij1

Example 2 Use alternating symbol to evaluate u, where u is a vector.


Solution 2 It can be easily verified that
(curlu)i = ( u)i =

ijk uk,j

ijk

uk
xj

(1.36)

or that
u=

1.1.3

ijk uk,j ei

(1.37)

General Cartesian Tensors

Let a physical quantity T be defined in a three dimensional Euclidean space,


such that for a given choice of the basis {ei } it is possible to specify T in terms
of an ordered set of 3n numbers denoted by tijk... containing a total of n indices.
Definition 1 If the components of T in the new vector basis {e0i } become t0ijk... ,
where
t0ijk... =

ip jq kr ...tpqr...

(1.38)

then T is called a Cartesian tensor of order n.


Note: Multiplication of a tensor T by a scalar produces another tensor
T of the same order
t0ijk... =

ip jq kr ...tpqr...

(1.39)

Therefore, n=0,1, 2 corresponds to a scalar, vector, and a matrix, respectively.

Tensor Properties
1. Two tensors of the same ordered can be added: A + B = tensor of the
same order as A.
2. Operation of addition of tensors is commutative and associative, i.e.,
A + B = B + A; (A + B) + C = A + (B + C).
3. Two tensors are equal if their dierence is the null tensor, i.e., A B = 0
or aijk... bijk... = 0.
Tensor Products
Let u and v be two vectors (tensors of order 1) and S and T be two tensors of
order 2. Then we have
u0i
s0ij

=
=

vi0 = ik vk
t0ij =
jm skm ;

ik uk ;
ik

ik jm tkm

(1.40)
(1.41)

Consider a set of nine numbers aij defined by the product


aij = ui vj

(1.42)

It follows that
a0ij = u0i vj0 =

ik jm uk vm

ik jm akm

(1.43)

which implies that A is a tensor of order 2.


Similarly, the numbers defined by
bijk = tij uk

(1.44)

obey the following transformation law


b0ijk = t0ij u0k =

ip jq tpq kr ur

ip jq kr bpqr

(1.45)

which states that B is a tensor of order 3 with 27 components (33 ). Also, the
set of 34 = 81 numbers defined by
cijkl = sij tkl

(1.46)

represents the components of a tensor of order four.


Definition 2 The products in Eqns. 1.42, 1.45 and 1.46 are called outer products of the tensors.
They can be extended to higher order tensors. An outer product between
two tensors generates a tensor whose order is the sum of the orders of the
corresponding tensors.
Definition 3 An inner product between two tensors is characterized by the presence of one or more of dummy indices.
6

The following are examples of inner products:


= ui vi
pi = tij uj
qik = sij tkj

(1.47)
(1.48)
(1.49)

It is easy to show that , p, and q are tensors of order zero, one , and two,
respectively.
Note: The order of the resulting tensor is always less in the inner products
than in the corresponding outer products.
Apparently
= aii
pi = bijj
qik = cijkj

(1.50)
(1.51)
(1.52)

where the quantities on the RHS of the last three equations are obtained by
putting two indices equal and then summing over this index. This operation is
called contraction. So, a scalar is obtained by contracting the second order
tensor A, the vector p is obtained by contracting the third order tensor B, etc.
In general, the contraction of a tensor of order n (>2) results in a tensor of
order n-2.
Symmetry of Tensors
Definition 4 Tensor W with components wij is symmetric if
wij = wji

(1.53)

wij = wji

(1.54)

and it is antisymmetric if
Note: For any antisymmetric tensor w11 = w22 = w33 = 0.
This definition can be extended to tensors of higher order, the symmetry
and antisymmetry being defined with respect to a particular pair of indices
Definition 5 Tensor W with components wijkl... is symmetric in the pair of
indices jk if
wijkl... = wjikl...
(1.55)
Symmetry and antisymmetry of a tensor are intrinsic properties, independent of coordinate system in which it is represented. For example, for a symmetric tensor W of order 2 we have
0
wij
=

ir js wrs

ir js wsr

is jr wrs

jr is wrs

0
= wji

(1.56)

Or in another words, if W is symmetric in coordinate system {xi }; i = 1, 2, 3,


it is also symmetric in the coordinate system {x0i }; i = 1, 2, 3.
7

The Quotient Rule


Theorem 1 Let W be an entity in {xi }; i = 1, 2, 3 coordinate system represented by an ordered nine quantities wij . Suppose for all vectors v the scalars
ui = wij vj

(1.57)

are components of a vector u. Then W is a second order tensor.


Proof. From the assumption we have that
0 0
0
u0i = wij
vj = wij

js vs

ir ur

ir wrs vs

(1.58)

from which it follows that


0
(wij

for all vs . Therefore,

js

0
wij

js

By multiplying the last equation by


0
wij

ir wrs )vs

=0

(1.60)

ir ts wrs

(1.61)

= tj it follows that

ts js

0
wit
=

(1.59)

we obtain

ts

ts js

Due to orthogonality relationship

ir wrs

=0

ir ts wrs

(1.62)

which completes the proof.


The proof extends to tensors of higher order. In general we have
Theorem 2 Let the components of a tensor of order m be uijk... and the components of a tensor of order n be vrst... . Then if relationship
uijk... = wijk... vrst...

(1.63)

holds for all uijk... and vrst... , then wijk... re components of a tensor of order
m+n.
Example 3 The set of scalars uijkm is such that for all second order tensors
with components ekm , the scalars ij = uijkm ekm are the components of a second
order tensor . Then, uijkm are the components of a fourth order tensor.
Solution 3
0ij = u0ijkm e0km = u0ijkm

kr ms ers

il jn ln

il jn uln rs ers

(1.64)

implies that
(u0ijkm
for all ers . Therefore,

kr ms

u0ijkm

kr ms

il jn uln rs )ers

=
8

il jn uln rs

=0

(1.65)
(1.66)

By multiplying the last result with


u0ijkm

tr os

kr tr ms os

Using the orthogonality relationships


u0ijto =

it follows that

il jn tr os uln rs

kr tr

= kt and

ms os

(1.67)
= mo we obtain

il jn tr os uln rs

(1.68)

which completes the proof.

1.1.4

Isotropic Tensors

Definition 6 A Cartesian tensor is said to be isotropic if its components are


identical regardless of how the axes of the coordinate system are rotated.
The following properties can be deduced for isotropic tensors:
1. There are no nontrivial isotropic tensors of order 1.
2. Any isotropic tensor of order 2 can be written as I.
3. The alternating symbol ijk is a component of a 3rd order isotropic tensor
and any 3rd order isotropic tensor can be written as .
4. If C is an isotropic tensor of order 4, then
cpqrs = pq rs + pr qs + ps qr

(1.69)

The first property is self evident. The second property follows from
0ij =

ir js rs

= ij

(1.70)

The third property can be shown as following:


0
ijk

=
=

ir js kt rst

+ i2 j3 k1 + i3 j1 k2
k2 i2 j1 k3 i3 j2 k1

i1 j2 k3

i1 j3

= det

i1

j1

k1

i2

j2

k2

i3

j3

k3

which completes the proof.

1.1.5

ijk

Tensor Fields

Definition 7 If the components wijk... of the tensor W are functions of the


coordinates {xi }, i = 1, 2, 3 of the points in given region, then W is called a
tensor filed or tensor function of position.
9

The main feature of a tensor field is ease of multiple partial derivatives of


the components wijk... . Usually, they are denoted by
wijk...
xp
2 wijk...
xp xq

= wijk...,p

(1.71)

= wijk...,pq

(1.72)

etc. For example,


,i

(1.73)

represents components of a gradient of a scalar field.

1.1.6

Tensor Gradient

Definition 8 In an open region R, let the components wijk... of tensor W of


order n be continuously dierentiable functions of {xi }, i = 1, 2, 3. Then,
wijk...,p

(1.74)

are the components of a tensor field of order n+1 called the tensor gradient of
W.
Proof. (of consistency). From
0
wrst...
=

it follows that

ri sj ... ri sj ...wij...

(1.75)

wij... xp
xp x0q

(1.76)

0
wrst...
=
x0q

ri sj ...

Since
xp =
we obtain that

0
wrst...
=
x0q

0
qp xq

ri sj ... qp

(1.77)
wij...
xp

(1.78)

Special Cases
For a scalar field ,i denotes the components of a 1st order tensor.
Thus grad = = ,i ei . Similarly ,ij are components of a second
order tensor, and ,ii = 2 .
For a vector filed f , fi,j denotes components of a 2nd order tensor and
fi,i = f =divf . Similarly, (Curlf )i = ijk fk,j or f = ijk fk,j ei .
Using tensor notation we can verify many vector identities. For example:
10

1. div(u v) = vCurlu uCurlv


2. Curl(u) =Curlu ugrad
3. CurlCurlu =grad(divu)2 u
To prove identity 1 we proceed as following. Let
w = u v = ijk ui vj ek

(1.79)

=
= vj jki ui,k ui ikj vj,k
= vj (Curlu)j ui (Curlv)i

(1.80)
(1.81)
(1.82)

Then
divw

Proof of identity 3 can be constructed in similar fashion. Namely, let


w =Curlu = u = ijk uk,j ei

(1.83)

Then
Curlw

=
=

ijk wk,j ei
kij

= ijk ( kmn un,m ),j ei


kmn (un,mj )ei

(1.84)
(1.85)

= im jn in jm

(1.86)

Using the so called equality


kij kmn

it follows that
Curl(Curlu) = ( im jn in jm )(un,mj )ei

(1.87)
2

= (uj,ij vj ui,jj )ei = ( u) u

1.2

(1.88)

The Divergence Theorem

Theorem 3 Let V denote a volume of a region space bounded by a piecewise


smooth surface S and its interior and let ui (x) be the components of a vector
filed in a fixed vector basis {ei } that span the space. Assume that the first partial
derivatives ui,j exist and are continuous in V . Then,
Z
Z
Z
Z
divudV =
u ndS
ui,i dV =
ui ni dS
(1.89)
V

where n is the outward unit normal to S at the surface element dS.


Proof. See Kellog, O. D. (1953). Foundations of Potential Theory, Dover,
New York, pp. 37-39.

11

1.2.1

Generalization to Tensors of Any Order

Theorem 4 Let us consider a tensor filed Ajkl... . Let the region V with boundary surface S be within the region of definition of Ajkl... . Assume that every
component of Ajkl... is continuously dierentiable. Then,
Z
Z

Ajkl... dV =
ni Ajkl... dS
(1.90)
V xi
S
Proof. For u =A(x)ei , i = 1, 2, 3, Eq. 1.89 implies that
Z
Z
A
dV =
Ani dS
V xi
S
for every Ajkl... . Therefore,
Z
V

Ajkl... dV =
xi

ni Ajkl... dS
S

Exercise 1 Show that


Z
Z
Z
Z
,i dV =
ni dS
dV =
ndS
V

(1.91)

Exercise 2
Z

ui,i =

Exercise 3
Z

ijk uk,j dV

1.3

ui ni dS

ijk uk nj dS

u=

u=

u ndS

(1.92)

(1.93)

n udS

Dyadics

Consider a vector v expressed in terms of the basis {ei } or in terms of the basis
{e0i }
v =vi ei = vi0 e0i
(1.94)
This direct representation can be extended to the higher order tensors.
Definition 9 Consider now a second-order tensor A with components aij , i, j =
1, 2, 3. We form the following expression
a = aij ei ej

(1.95)

where the base vectors are juxtaposed in a specific order and the summation
convention applies. Then the last equation defines a dyadic.
12

Since
e0i =

ij ej

& ei =

0
ji ej

(1.96)

Eq. 1.95 implies that


a = aij
However, aij

ki mj

0
0
ki ek mj em

= (aij

0 0
ki mj )ek em

(1.97)

= a0km and thus


a = aij ei ej = a0km e0k e0m

(1.98)

These expressions are similar to those for v in Eq. 1.94, except that the base
vectors ei and e0i have been replaced by the juxtaposed pairs ei ej and e0i e0j .
Consequently Eq. 1.98 can be interpreted as a direct representation of the
second-order tensor A.
Algebraic operations can be carried out directly with a provided certain rules
of operations on the pair ei ej are followed.
Since v =vi ei = vi0 e0i it follows that
(ek a) em = (ek aij ei ej ) em = aij ki jm = akm

(1.99)

Note that
(em a) ek = amk

(1.100)

Therefore, the ordering of ei ej in Eq. 1.98 is critical in carrying out


algebraic operations on a and in general it cannot be changed arbitrarily.
All the notions associated with the second-order tensor A (symmetry,
transpose, orthogonality, eigenvalues, and principal axes) may be used for
dyadics.
Special dyadics
For vectors u and v:
ui vj ei ej dyad

(1.101)

The dyadic corresponding to the unit tensor I is


i = ij ei ej = e1 e1 + e2 e2 + e3 e3

(1.102)

is called unit dyadic.


The following results can be proved:
ui
(a u) v
v(a u)
(ua) v

=
=
=
=
13

iu=u
a (u v)
(va) u
u (a v)

(1.103)
(1.104)
(1.105)
(1.106)

x3
(r,,z)

z
O

x2

x1
Figure 1.2: Cylinrical coordinate system.

For example, the proof of the identity ui = i u = u can be constructed as


following
ui = (uk ek ) ij ei ej = uk ki ij ej = uj ej = u
i u = ij ei ej uk ek = ij ei jk uk = ui ei = u

(1.107)
(1.108)

Similar proofs can be constructed for the other identities.

1.4

Vector Identities in Cylindrical Coordinates

Lets consider cylindrical coordinate system {r, , z} with unit bases {er , e , ez }
(Fig. 1.2)
The corresponding Cartesian components are given by
x1 = r cos ;

x2 = r sin ;

x3 = z

(1.109)

and an element of an arc of a curve C can be written as


ds2 = dr2 + (rd)2 + dz 2

(1.110)

Consequently it follows that


f (r, , z) = (er

+ e
+ ez )f
r
r
z

(1.111)

Recall that unit tangent vector along a space curve is given by


t=

x
s

14

(1.112)

where s denotes a parameter along the curve. The position vector x can be
written as
x =x1 e1 + x2 e2 + x3 e3 = r cos e1 + r sin e2 + ze3

(1.113)

then from Eqns. 1.112 and 1.113 it follows that


er

ez

x
= cos e1 + sin e2
r
x
= sin e1 + cos e2
r
x
= e3
z

(1.114)

Since ei , i = 1, 2, 3 are constant vectors we obtain the following


er
r
e
r
ez
r

= 0;
= 0;
= 0;

er
= e ;

e
= er ;

ez
= e ;

er
=0
z
e
=0
z
ez
=0
z

(cy8)

Let u be a vector with components (ur , u , uz ) along (er , e , ez ), i.e.


u =ur er + u e + uz ez

(1.115)

Then

+ e
+ ez )(ur er + u e + uz ez )
r
r
z
ur
u
uz
= er (
er +
e +
ez )
r
r
r
1
ur
u
uz
+ e (
er + ur e +
e u er +
ez )
r

ur
u
uz
+ez (
er +
e +
ez )
z
z
z

u = (er

(1.116)

from which it follows that


u =

ur
1
u
uz
er er + (ur +
)e e +
ez ez
r
r

z
1 uz
u
ur
uz
+
e ez +
ez e +
ez er +
er ez
r
z
z
r
u
1 ur
+
er e + (
u )e er
r
r

(1.117)

Consequently, we have that


u=

u
ur
1
uz
+ (ur +
)+
r
r

z
15

(cy10)

x3
(R,,)
R

x2

x1
Figure 1.3: Spherical coordinate system.
Note in order to calculate u simply perform the dot products between
each pair of unit vectors in Eq. 1.117.
Similarly, by noting that e ez = er , etc. ... it follows that
u =(er

+ e
+ ez ) (ur er + u e + uz ez )
r
r
z

(1.118)

or that
u = er (

1 uz u
ur uz
u u 1 ur

) + e (

) + ez (
+

) (1.119)
r
z
z
r
r
r
r

Example 4 2 f (r, , z) =?.


Solution 4 From
2 f = f

(1.120)

u =f

(1.121)

and
using Eq. 1.111 we obtain
2 f =

1.5

2f
1 f
2f
1 2f
+
+
+
r2
r r
r2 2
z 2

(1.122)

Vector Identities in Spherical Coordinates

Lets consider spherical coordinate system {R, , , } with unit bases {eR , e , e }
(Fig. 1.3)
Then corresponding Cartesian coordinates are given by
x1 = R sin cos ;

x2 = R sin sin ;
16

x3 = R cos

(1.123)

Then the element of an arc of a curve is defined by


ds2 = dR2 + (Rd)2 + (R sin d)2
and
f (R, , ) = (eR

1
+ e
+ e
)f
R
r
R sin

(1.124)
(1.125)

Therefore we have that


x = sin cos e1 + R sin sin e2 + R cos e3

(1.126)

and that
eR

x
= sin cos e1 + sin sin e2 + cos e3
R
1 x
= cos cos e1 + cos sin e2 sin e3
R
1 x
= sin e1 + cos e2
R sin

(1.127)

eR
eR
= e ;
= sin e

e
e
= eR ;
= cose

e
e
= 0;
= sin eR cos e

(1.128)

Therefore,
eR
R
e
R
e
R

= 0;
= 0;
= 0;

Based on these results we have that

1
+ e
+ e
)(uR eR + u e + u e )
u =(eR
R
r
R sin

(1.129)

which leads to
uR
u
u
eR +
e +
e )
R
R
R
e uR
u
u
(
eR + uR e +
e u eR +
e )
R

e
u
uR
+
(
eR + uR sin e +
e + u cos e
R sin

u
+
e sin u eR cos u e )

u = eR (

which after some algebra becomes


u =

uR
1
u
1
u
eR eR + (uR +
)e e +
(sin uR + cos u +
)e e
R
R

R sin

1 u
1
1
u
uR
+
e e +
(
cos u )e e +
(
sin u )e eR
R
R sin
R sin
u
u
1 uR
+
(1.130)
eR e +
eR e + (
u )e eR
R
R
R
17

From the last result we obtain that


u=

uR
1 u
1 u
2
+ uR + (
+ cot u ) +
R
R
R
R sin

(1.131)

Similarly, it follows that


1 u
1 u
(
+ cot u
)
R
sin
1 uR u u
+e (

)
R sin
R
R
u
u
1 uR
+e (
+

)
R
R
R

u = eR

or

e
1 R
u=
R
R sin
uR

Re

R sin e

Ru

R sin u

Example 5 Evaluate 2 f in spherical coordinates.

(1.132)

(1.133)

Solution 5 Using the fact that 2 f = f and Eqns. 1.125 and 1.131 it
follows that
2 f =

1.6

2f
2 f
f
1 2f
1 2f
+
(
+
cot

+
+
R2 R R R2 2

sin2 2

(1.134)

Eigenvalue Problem for Real Symmetric Tensors of Order Two

Definition 10 Let A be a real symmetric tensor of order 2 with components


aij in the vector basis {ei }. Let v be a vector with components vi in {ei }. Then
the eigenvalue problem for A is defined by
Av =v aij vj = vi

(1.135)

for being a scalar.


Equation 1.135 can be written as
(AI)v = 0 (aij ij )vj = 0

(1.136)

The system 1.136 has nontrivial solution if and only if

or

det(AI) =0

(1.137)

a12
a13
a11
a22
a23 = 0
det a21
a31
a23
a33

(1.138)

18

Equation 1.138, called the characteristic equation, is cubic in and it may be


written as
D() = 3 IA 2 + IIA IIIA = 0
(1.139)
where the invariants IA , IIA , and IIIA are defined by
IA
IIA
IIIA

= aii
1
=
(aii ajj aij aji )
2
= det A = ijk a1i a2j a3k

It is easy to show that IA = trace(A) and that

a11 a12
a11 a13
a22
IIIA = det
+ det
+ det
a21 a22
a31 a33
a32

(1.140)

a23
a33

(1.141)

The roots of the characteristic equation are called the eigenvalues of A and
the corresponding nontrivial solutions v of Av =v are called the eigenvectors of A. There are three eigenvalues k , k = 1, 2, 3 of A and corresponding
eigenvectors are denoted by v(k) , i.e.,
Av(k) =k v(k) ; k = 1, 2, 3.

(1.142)

Clearly, if v is solution of Eq. 1.136, so is v, where denotes a scalar.


Consequently, the length of the eigenvectors is arbitrary. We shall normalize
them to unit vectors.
Theorem 5 If A is a real symmetric second-order tensor then all eigenvalues
of A are real.
Proof. From
aij vj
aij vj

= vi
= vi

(1.143)
(1.144)

where an overbar denotes the complex conjugate it follows that


aij vi vj
aij vj vi

= vi vi = aij vj vi
= vi vi

(1.145)
(1.146)

By subtracting the last two equations we obtain


( )vi vi = 0

(1.147)

Since vi vi 6= 0, it follows that = .


Theorem 6 If A is a real symmetric second-order tensor then the eigenvectors
corresponding to two distinct eigenvalues of A are orthogonal.

19

Proof. Suppose that the two distinct eigenvalues are denoted by 1 and 2 .
Then from
(1)

aij vj

(1)

(1.148)

(2)
2 vi

(1.149)

= 1 vi

(2)
aij vj

(1) (2)

= aij vi vj

(2) (1)

= aij vi vj

it follows that
1 vi vi
2 vi vi
and thus

(1) (2)

(1 2 )vi vi
(1) (2)

Since 1 6= 2 it follows that vi vi

= 0.

20

(1) (2)

(1.150)

(1) (2)

(1.151)

=0

(1.152)

Chapter 2

Strain Tensor
Let us assume that a point of an elastic body before deformation is specified by
P (a1 , a2 , a3 ). At a later instant of time the body is deformed and the point P
moved to Q(x1 , x2 , x3 ). We are assuming the change of body to be continuous
and the transformation of P to Q to be one-to-one and that it can be specified
as
xi = xi (a1 , a2 , a3 ); i = 1, 2, 3
(2.1)
i.e., coordinates of a point in deformed state can be expressed in terms of coordinate of the point in undeformed state. We also assume that this has a unique
inverse
ai = ai (x1 , x2 , x3 ); i = 1, 2, 3
(2.2)
or that coordinates of a point in undeformed state can be expressed in terms of
coordinates of the same point in deformed configuration.
We are interested in describing the strain of the body fibers, i.e., with stretching and distortion of the body. If. P P 0 P 00 are three closely spaced points forming
a triangle in the original configuration, and if they change to QQ0 Q00 in deformed
configuration, the change of the area and the angles of the triangle is completely
defined if we know the change in the length of the sides.
Consider now an infinitesimal element connecting P (a1 , a2 , a3 ) with P 0 (a1 +
da1 , a2 + da2 , a3 + da3 ). The square of length ds0 of P P 0 is given in the original
configuration by
ds20 = dai dai
(2.3)
Since P moves to Q(x1 , x2 , x3 ) and P 0 to Q0 (x1 + dx1 , x2 + dx2 , x3 + dx3 ) it
follows then that the square of length ds of QQ0 in deformed configuration is
given by
ds2 = dxi dxi
(2.4)
Using Eq.2.1 we obtain that
ds2 =

xi
xi xi
xi
daj
dak =
daj dak
aj
ak
aj ak

21

(2.5)

Q
P

Q(x1,x2,x3)

P(a1,a2,a3)

X3

P
X2

O
X1

Figure 2.1: Deformed and undeformed states of an elastic body.


and consequently
ds2 ds20 = (

xi xi
x x
jk )daj dak = (
jk )daj dak
aj ak
aj ak

(2.6)

Similarly, by using Eq.2.2 and


ds20 =

a a
dxi dxj
xi xj

we obtain
ds2 ds20 = ( ij

a a
)dxi dxj
xi xj

(2.7)

(2.8)

Thus we define Green strain tensor


Eij =

x x
1
jk )
(
2
aj ak

(2.9)

1
a a
)
( ij
2
xi xj

(2.10)

and Almansi strain tensor


eij =
Therefore, we can write
ds2 ds20 = 2Eij dai daj

(2.11)

ds2 ds20 = 2eij dxi dxj

(2.12)

Using quotient rule it follows that Eij and eij are components of secondorder tensors.
22

Remark 1 It should be noted that Green tensor is defined in terms of undeformed coordinates ai while Almansi strain tensor is defined in terms of deformed coordinates xi .
Since components of displacement vector are given by
ui = xi ai

(2.13)

we can obtain dierent forms of the Green and Alamansi strain tensors which
are more commonly used
Eij

=
=
=

1
u
u
+ i )(
+ j ) ij ]
[ (
2
ai
aj
u
1
u u
u
+
j + i
+ i j ) ij ]
[ (
2
ai aj
ai
aj
uj
u u
1 ui
+
+
)
(
2 aj
ai
ai aj

(2.14)

Similarly,
eij =

1 ui
uj
u u
+

)
(
2 xj
xi
xi xi

(2.15)

In unabridged notation (x, y, z) for (x1 , x2 , x3 ), (a, b, c) for (a1 , a2 , a3 ), and


(u, v, w) for (u1 , u2 , u3 ) we have that components of the Green strain tensor
(Lagrangian strain tensor) is given by
Eaa
Eab

u 1 u 2
v
w 2
+ [( ) + ( )2 + (
) ]
a 2 a
a
a
1 u v
2u
2v
2w
=
[
+
+(
+
+
)]
2 b
a
ab ab ab
..
.
=

(2.16)

while the components of the Almansi strain tensor ( Eulerian strain tensor) are
given by
exx
exy

u 1 u 2
v
w 2
[( ) + ( )2 + (
) ]
x 2 x
x
x
1 u v
2u
2v
2w
=
[
+
(
+
+
)]
2 y
x
xy xy xy
..
.
=

(2.17)

Remark 2 u, v, and w are considered as functions of a, b, a, the position points


in the body in undeformed configuration when the Lagrangian strain tensor is
evaluated whereas they are considered as functions of x, y, z (the positions points
in deformed configuration) when Eulerian strain tensor is evaluated.

23

If the components of displacement field ui are such that their first derivatives
are so small that the squares and the product of their partial derivatives of ui
are negligible, then eij reduces to Cauchys infinitesimal strain tensor
eij =

1 ui
uj
1
+
) = (ui,j + uj,i)
(
2 xj
xi
2

(2.18)

which in unabridged notation gives


exx
exy
eyz

v
w
; ezz =
y
z
1 u v
1 u w
+
); exz = ezx = (
+
)
= eyx = (
2 y
x
2 z
x
1 v w
= ezy = (
+
)
2 z
y
=

u
;
x

eyy =

(2.19)

Remark 3 In the infinitesimal displacement theory, the distinction between the


Lagrangian and Eulerian strain tensor disappears. Namely
=

eij

=
=
=

2.1
2.1.1

1 ui
uj
1 ui ak
uj ak
+
)= (
+
)
(
2 xj
xi
2 ak xj
ak xi
1 ui xk
uk
uj xk
uk
(

)+
(

)]
[
2 ak xj
xj
ak xi
xj
1 ui
uk
uj
uk
( kj
)+
( ki
)]
[
2 ak
xj
ak
xj
1 ui
uj
+
] = Eij
[
2 aj
ai

Geometric Interpretation of Infinitesimal Strain


Components
Component exx

For that purpose we examine an elastic element of length dx and dy along the
x- and y-axis, respectively. We assume that u
x > 0. Then, if the element is
extended in the x-direction uniformly along the y-axes we have
ds0

= dx

ds = dx + u +

u
u
dx u = (1 +
)dx
x
x

which implies that


ds2 ds20 = 2eij dxi dxj = 2exx dx2

24

y
u+uxdx

dx

Figure 2.2: Elastic element streched in horizontal direction.


or

2exx
2
ds ds0
=
ds0 =
exx
ds0
ds + ds0
2 + u
x

Using the assumption of small strain yields then


exx =

ds ds0
ds0

or that the component exx measures the change of length per unit length of a
fiber parallel to the x-axis. Consequently, the component exx describes extension
(compression) of a material fiber. Similar conclusions follow for the components
eyy and ezz .

2.1.2

Component exy

To see the geometrical interpretation of this component of the starin tensor,


let us consider again the small rectangle of the material which undergoes the
following deformations
Apparently, before deformation the angle ](OP , OP 0 ) = /2. After deformation we have that
u
tan =
y
v
tan =
x
Suppose that tan > 0 for increasing in clockwise (CW) direction, while
tan > 0 for increasing in CCW direction. For small strains we may take
that tan and tan . Therefore,
exy =

1 u v
1
(
+
) = ( + )
2 y
x
2
25

Q
Q(u+uydy,v+vydy)

dy
O(u,v)
O

dx

Q(u+uxdx,
v+vxdx)
x

Figure 2.3: Shearing strain interpretation.


which implies that the strain component exy is equal to one half of the amount
(in radians) the right angle between two fibers is diminished due to deformation.
In engineering, the strain components eij ; i 6= j are called the shearing
strains.

2.1.3

Simple Shear

Consider a case v/x = u/x = 0, i.e. u and v remain unchanged in the


x-direction. Also, we assume that u/y > 0. Then the deformed element looks
like
and we have that
1 u
exy =
2 y

2.1.4

Rotation Vector

Consider a small element dxdy with u/y > 0 and v/x > 0. Then,
we have that
v
>0
x
u
=
>0
y

tan =
tan

where denotes an angle for which OP rotates about the z-axis and represents
an angle for which OP 0 rotates about the z-axes. Consequently,
z =

1 v
u
(

) measure of rotation
2 x y

26

y
P

dy

x
u

dx

Figure 2.4: Simple shear.

Q(uydy,
vydy)

dy

Q(uxdx,vxdx)

dx

Figure 2.5: Definition of rotation vector.

27

Remark 4 If there is no shear strain then exy = 0 and thus


v
u
=
y
x
Consequently, z = , which implies rigid-body-rotation (i.e. ](OQ, OQ0 ) =
/2).
In general we have that rotation vector is defined by
1
= u
2

2.1.5

(2.20)

Rotation Tensor

Definition 11 For infinitesimal displacement field ui (x1 , x2 , x3 ) the antisymmetric second order tensor , defined by
ij =

1
(uj,i ui,j )
2

(2.21)

is called the rotation tensor.


It is interesting to observe the following
1
2

ijk jk

1
2

ijk uk,j

1
( u)i = i
2

(2.22)

or that the rotation tensor is related to the rotation vector. In addition we have
lmi i

1
2

ijk lmi jk

1
[ jl km jm kl ]jk = lm
2

Therefore,
ij =

2.1.6

ijk k

(2.23)

Physical Interpretation of Rotation Vector and Rotation Tensor

Theorem 7 Vanishing of the symmetric strain tensor is necessary and sucient condition for a neighborhood of a particle to move like a rigid body.
Proof. Necessary condition. Whenever neighborhood of a particle moves
as a rigid body , then ds = ds0 .Thus
ds2 ds20 = 2eij dxi dxj = 0
for all dxi dxj . Therefore, eij = 0.
Sucient condition. When eij = 0, then 2eij dxi dxj = 0 and thus ds = ds0 .

28

Theorem 8 When the strain tensor vanishes at point P, the infinitesimal rotation of the RB motion of a neighborhood of P is given in terms of the rotation
vector .
Proof. Lets consider point P 0 in the neighborhood of P . Let P (xi ) and
P (xi + dxi ). Then, the relative displacement of P 0 with respect to P is given
by
0

dui

ui
dxj
xj
1
1
=
(ui,j + uj,i )dxj + (ui,j uj,i )dxj
2
2
= eij dxj ij dxj = ijk k dxj
=

or that
du = dx
The last results indicates that the relative displacement of P 0 with respect to P
is equivalent to an infinitesimal rotation about an axis through P in direction
of .

2.2

Compatibility and Strain Components

The question arises of how to determine the displacements ui , i = 1, 2, 3 when


the components of strain tensor eij are known or how to integrate the dierential
equations
1
eij = (ui,j + uj,i )
(2.24)
2
Apparently, there are three unknowns and six equations. Consequently,
Eqns.2.24 will not have a single solution in general if the functions eij are specified arbitrarily. Therefore, one may suspect that a solution may exist only if
the functions eij satisfy certain conditions.
Since strain components only involve relative positions of points in the body,
and since RB-motion correspond to zero strain we expect to have the solution
ui to within a RB-motion.
If the stearins are prescribed arbitrarily we may encounter the following
situations
For a single valued continuous solution to exist, the points C and D must
meet perfectly in the strained configuration. In order to achieve that the strain
components must satisfy certain conditions. They are to be satisfied by the
strain components eij and can be obtained by elimination of ui from Eq. 2.24.

29

Figure 2.6: Fibers in a body before deformation.

Figure 2.7: Fibers in a body which may result from arbitrarily prescribed strains.

30

Therefore,
eij,kl

ekl,ij

eik,jl

ejl,ik

1
(ui,jkl + uj,ikl )
2
1
(uk,lij + ul,kij )
2
1
(ui,kjl + uk,ijl )
2
1
(uj,lik + ul,jik )
2

and subsequently
eij,kl + ekl,ij eik,jl ejl,ik = 0;

i, j, k, l = 1, 2, 3

(2.25)

These equations known as the compatibility equations were first derived by St.
Venant (1860).
In general, Eq. 2.25 represents 34 = 81 equations. However, due to symmetry of the strain tensor there are only six independent equations. In unabridged
notation they are listed as
2 exx
yz
2 eyy
xz
2 ezz
xy
2 exy
2
xy
2 eyz
2
yz
2 exzx
2
zx

=
=
=
=
=
=

eyz
ezx exy
(
+
+
)
x
x
y
z

ezx exy
eyz
(
+
+
)
y
y
z
x

exy
eyz
ezx
(
+
+
)
z
z
x
y
2 exx 2 eyy
+
y 2
x2
2
2 ezz
eyy
+
z 2
y 2
2
2 exx
ezz
+
x2
z 2

(2.26)

In two-dimensional case we have that u = u(x, y); v = v(x, y); w 0. Therefore, ezx = ezy = ezz = 0 and ()/z 0. In that case the compatibility
equations reduce to a single equation
2

2 exy
2 exx 2 eyy
+
=
xy
y 2
x2

(2.27)

Example 6 2D Case. Suppose that the strain filed is specified by exx = 3x +


sin y, eyy = x3 , exy = 12 (x cos y + ax2 y). Determine a suitable value for a and
then calculate displacement filed u and v.
Solution 6 Compatibility equation (2.27) implies that
sin y + 2ax = sin y + 6x
31

and consequently a = 3. Now


exx =

u
= 3x + sin y
x

which implies that


u=

3 2
x + x sin y + C1 (y)
2

Similarly, from
eyy =

v
= x3
y

we get
v = x3 y + C2 (x)
Consequently, from
exy =

1 u v
1
(
+
) = (x cos y + 3x2 y)
2 y
x
2

we have that
1
1
(x cos y + 3x2 y + C10 (y) + C20 (x)) = (x cos y + 3x2 y
2
2
from which it follows that
C10 (y) = C20 (x)
for all x and y. Therefore,
C20 (x) = C10 (y) = const = A
and
C1

= Ay + C
C2 Ax + B

where A, B, and C are constants. Finally, we obtain


3 2
x + x sin y Ay + C
2
v(x, y) = x3 y + Ax + B

u(x, y) =

32

(2.28)

Chapter 3

Stress Tensor
Consider an elastic body B subjected to a system of external forces Pi , i =
1, ..., n (see Fig. 3.1)
Suppose we cut the body into two parts B1 and B2 by a plane so that
B = B1 B2 . In order for separate parts to be in equilibrium has to apply to
parts B1 and B2 the forces F and F, respectively which are the resultant
of internal forces acting on the plane of the section S. Consider now an
element of area S with corresponding internal force resultant F. Then, the
stress vector or traction is defined by
F
(3.1)
S
Since Tn will depend upon the orientation of the surface S (specified by the
unit normal vector n), superscript n is introduced with the traction vector.
Choice of surface S is arbitrary. It is of interest, however, to consider some
special cases in which the surface S = Sk is parallel to one of the coordinate planes. Let the normal to Sk be in the positive direction of the
xk -axis, and let the stress vector acting on Sk be denoted by Tk with components T1k , T2k , and T3k along the axes x1 , x2 , and x3 ,respectively. For this special
case the following convention is used
Tn lim

S0

Tjk = kj ;

k, j = 1, 2, 3

(3.2)

Therefore, for the surface perpendicular to x1 -axis we have stress vector components 1j , for the surface perpendicular to x2 -axis we have stress vector components 2j , and for the surface perpendicular to x3 -axis we have stress vector
components 3j , j = 1, 2, 3. Graphically, the components of a stress vector can
be depicted as in Fig. 3.3
Thus, ii , i = 1, 2, 3 (no summation) are defined as normal stresses, while
ij , i 6= j are known as shear stresses.
Definition 12 Sign Convention: Positive normal stress vector points away
from the material. Positive shear stress ij , i 6= j means the stress vector is in
33

P1

P1

1
S
Pn

x3
O

x2

x1
Figure 3.1: An elastic body subjected to a system of forces.
the positive direction of xj axis when the positive direction of the xi axis point
out of the body.

3.1

Symmetry of the Stress Tensor

Let us consider an infinitesimal cube of size dx1 dx2 dx3 subjected to stresses ij
and a body force b (see Fig. 3.4)
P
Since the box is in equilibrium, then Mx1 = 0, where x1 is an axis parallel
with x1 and going through the centroid of the cube. Thus
dx2
dx2
23
dx2 )dx1 dx3
+ ( 23 +
2
x2
2
32
dx3
dx3
dx3 )dx1 dx2
32 dx1 dx2
( 32 +
2
x3
2

23 dx1 dx3

must be zero. By taking the limit as V 0 and dxi 0 we obtain


1 23
1 32
dx2 32 lim
dx2 = 0
dx2 0 2 x2
dx2 0 2 x2

23 + lim
which leads to

23 = 32
34

(3.3)
(3.4)

D F
n

D S
Figure 3.2: The resultant internal force on an infinitesimal area.

35

33

s
s

31

32

23

13

21

12

11

x3
x2
x1
Figure 3.3: The components of stress vector.

36

22

x3
s

+
Ds

33

s
s
s
dx3

31

+
Ds
s

23

+
Ds

r b2 s
s

Ds
21+

s+D
2

21

x2

31

32

dx2

23

r b3
21

x1

32

s +Ds 23
31

r b1
2

32

dx1
s

33

Figure 3.4: An infinitesimal box of material in equilibrium.


P
P
Mx3 = 0 results in 12 = 21 .
Similarly,
Mx2 = 0 provides 13 = 31 and
Therefore, provided there are no internal moments proportional to a volume we
have that
ij = ji ; ij = 1, 2, 3
(3.5)
or that the stress tensor is symmetric.

3.2

Equations of Equilibrium

For equilibrium
P of an infinitesimal
P box (see Fig. 3.4) we must have that sum of
the forces
F = 0. Thus from
Fx1 = 0 it follows that
11
dx1 )dx2 dx3
x1
21
21 dx1 dx3 + ( 21 +
dx2 )dx1 dx3
x2
31
31 dx1 dx2 + ( 31 +
dx3 )dx1 dx2 + b1 dx1 dx2 dx3
x3

11 dx2 dx3 + ( 11 +

must be zero. By taking the limit limV 0 the last equation implies that
11 21 31
+
+
+ b1 = 0
x1
x2
x3
37

x2

D S
s

n
s

31

x1
21

x3
Figure 3.5: Traction boundary conditions.
P
P
Similar equations can be obtained for
Fx2 = 0 and
Fx2 = 0 which can be
summarized as the equations of equilibrium
ij,j + bi = 0;

i, j = 1, 2, 3

(3.6)

or equivalently
div + b = 0

3.3

(3.7)

Boundary Conditions

Consider a segment of an elastic body with surface S subjected to a surface


traction Tn (Fig.3.5). Unit normal on S is n.
Then projection of S on x1 x2 -plane is given by
S3 = S cos ](n,x3 ) = Sn3
Similarly it follows that
S2
S1

= Sn2
= Sn1

38

p0
a

S1

S2

Figure 3.6: An elastic wedge problem.


For equilibrium we must have
T1n S

Fx1 = 0 or

= 11 S1 + 12 S2 + 13 S3 + b1 V
= ( 11 n1 + 12 n2 + 13 n3 )S + b1 V

By taking the limit lim S 0 and noting that limxi 0 V /S = 0 we


obtain
T1n = 11 n1 + 12 n2 + 13 n3
Similarly, we have that
T2n
T3n

= 21 n1 + 22 n2 + 23 n3
= 31 n1 + 32 n2 + 33 n3

or
Tin = ij nj

(3.8)

The last result represents the Cauchy stress formula which relates external tractions to internal stresses. Since Tn and n are vectors, is a tensor of order
2.
Example 7 Boundary conditions for an elastic wedge (Fig. 3.6)
Solution 7
22 (x, 0) = p0 ; x S1
11 sin + 12 cos = 0; x S2
12 sin + 22 cos = 0; x S2

39

3.4

Plane State of Stress

Definition 13 State of stress in which 33 = 31 = 32 0 is called plane


state of stress in the plane x1 x2 .
According to transformation law 0ij = ir js rs , the transformation matrix
is easily satisfied by single angle

cos sin 0
L = sin cos 0
0
0
1

Then it is easily shown that


0xx
0yy
0xy

xx + yy
xx yy
+
cos 2 + xy sin 2
2
2
xx + yy
xx yy
=

cos 2 xy sin 2
2
2
xx yy
=
sin 2 + xy cos 2
2
=

(3.9)

Remark 5 Sum of normal stresses remains unchanged (an invariant) in different coordinate systems, i.e.,
xx + yy = 0xx + 0yy

(3.10)

Case 1 0xy = 0. Then we have that


tan 20 =
Since

thus 0xy

2 xy
xx yy

0xx
= 2 0xy

= 0 implies extremum in normal stresses. By using


cos 20
sin 20

= (1 + tan2 20 )1/2
= tan 20 (1 + tan2 20 )1/2

(3.11)

(3.12)

(3.13)
(3.14)

we obtain from Eqns.3.9 that


r
xx + yy
xx + yy 2
= I =
+ (
) + 2xy
2
2
r
xx + yy
xx + yy 2
0
yy (0 ) = II =
(
) + 2xy
2
2

0xx (0 )

(3.15)
(3.16)

where I and II are maximum and minimum normal stresses, respectively.


These stresses are known as the principal stresses.
40

Case 2 max 0xy . Then

0xy
=0

which implies that


tan 2 =

xx yy
2 xy

(3.17)

from which it follows that


0xy ( ) = 0xy max =

Since
tan 20 =

xx + yy 2
) + 2xy
2

(3.18)

1
tan 2

we have that
= 0 45o

(3.19)

which states that the directions of the principal stresses are bisected by
directions of maximum shear stresses.

3.5

Linear Momentum and the Stress Tensor

Suppose we remove from a body a closed region V + V . The surface V is


subjected to a distribution of surface tractions Tn (x,t). Each mass element of the
body may be subjected to a body force per unit mass f (x,t). Then the principle
of balance of linear momentum states that: Instantaneous rate of change of the
linear momentum of a body is equal to resultant external force acting on the
body at that time, or
Z
Z
Z
u

dV =
f dV +
Tn dS
(3.20)
t V t
V
V
Note: dV = dm is the particle mass. Also, Eq. 3.20 is based on a Lagrangian description, and V and S move with the particle. Since the particle
mass is constant in time we obtain from 3.20 that
Z
Z
Z
2u
2 dV =
f dV +
Tn dS
(3.21)
t
V
V
V
Using Cauchy stress formula it follows than that
Z
Z
Z

ui dV =
fi dV +
ij nj d
V
ZV
ZV
fi dV +
ij,j dV
=
V

or

( ij,j + fi
ui )dV = 0
41

(3.22)

Since V may be an arbitrary part of the body, we get that whenever the integrand in Eq. 3.22 is continuous that
ij,j + fi =
ui

(3.23)

which is Cauchys first law of motion.

3.6

Balance of Angular Momentum

The principle of balance of angular momentum states: Time rate of change of


angular momentum about O equals to the moment of the forces about O acting
in V. Thus
Z
Z
Z

x u,t dV =
x f dV +
x Tn dS
(3.24)
t V
V
V
Since (dV )/t = u,t u,t = 0 we obtain that
Z
Z
Z
klm xl u
m dV =
klm xl fm dV +
V

n
klm xl Tm dS

(3.25)

Using the Gauss theorem and equation of motion we obtain for the surface
integral
Z
Z
Z
n
=
klm xl Tm dS
klm xl mr nr dS =
klm (xl mr ),r dV
V
V
ZV
um fm )]dV
(3.26)
=
klm [ ml + xl (
V

Substituting Eq. 3.26 into Eq. 3.25 yields that


Z
klm ml dV = 0
V

or that
klm ml

which implies that


ml = lm
the stress tensor is symmetric.

3.7

Principal Stresses

Definition 14 In general state of stress, the stress vector Tn acting on a surface with outer normal n depends on n. We are asking in what direction n
the stress vector becomes normal to the surface on which there are no shearing
stresses (Fig.3.7). Such surface will be called the principal plane, its normal
the principal axis, and its value of normal stress acting on the principal plane a
principal stress.
42

T
n

n
S*
S

Figure 3.7: The principal stress.


Therefore, along the principal axes we have
Tn = n
or that
ij nj = ni

(3.27)

which is an eigenvalue problem. This can be written in the form


( ij ij )nj = 0

(3.28)

Condition for a nontrivial solution of Eq. 3.28 requires that


det(I) =0

(3.29)

which is the characteristic equation of the problem. The last result can be
written in the form
(3.30)
3 I 2 + II III = 0
where
1
I
II
III

= 1 ; 2 = 2 ; 3 = 3
= ii
1
=
( ii jj ij ij )
2
= det
43

(3.31)

Thus in the coordinate system {ei } the stress tensor is real and symmetric.
Then, in the coordinate system {e0i }, where the basis vectors are along the
principal axes (the eigenvectors of Eq. 3.28) {e0i }, the stress tensor becomes
diagonal, i.e.,

1 0
0
0 = 0 2 0
(3.32)
0
0 3

In that case the stress tensor invariants become


I
II
III

= 1 + 2 + 3
= 1 2 + 1 3 + 2 3
= 1 2 3

Example 8 The stress tensor is defined by

3 5 8
= 5 1 0
8 0 2

(3.33)

Determine the principal stresses and the principal directions. Write down the
value of stress invariants.
Solution 8 The stress invariants are
I
II

III

= ii = 3 + 1 + 2 = 6
1
( ii jj ij ij )
=
2
= 11 22 + 11 33 + 22 33 212 213 223
= 3 1 + 3 2 + 1 2 52 82 02 = 78
= det = 108

Corresponding characteristic equation is given by


3 I 2 + II III = 0
or
3 62 + 78 + 108 = 0
which produces
1
2
3

= 11.8242
= 1.2848
= 7.1090

The principal directions: For = 1 we have


(1 I)n(1) = 0
44

Tn
nt

n
s

nn

Figure 3.8: Stresses on an oblique plane.


which results in
n(1)

0.7300
= 0.3372
0.5945

Similarly, the last two eigenvectors are calculated to be

0.480
0.6818
n(2) = 0.8424 ; n(3) = 0.4204
0.5368
0.5988

Note that the eigenvectors are normalized so that | n(s) |= 1, s = 1, 2, 3.

3.8

Normal and Shearing Stresses on an Oblique


Plane

The normal stress nn on the plane P is the projection of the stress vector Tn
in the direction of the unit normal n of plane P (see Fig. 3.8). Thus
nn = Tn n = ij nj ni

45

(3.34)

200 psi

600

a
100 psi

100 psi
a

x2
200 psi
O

x1
Figure 3.9: A skewed plate problem.

or
nn

= 11 n21 + 22 n22 + 33 n23


+2 12 n1 n2 + 2 13 n1 n3 + 2 23 n2 n3

(3.35)

Similarly, the shearing stress


p
nt = Tn t = | Tn |2 2nn

(3.36)

Example 9 The skewed plate of unit thickness is loaded uniformly along the
sides of the plate as shown by Fig. 3.9. Determine the elements of the stress
tensor and normal stress on a plane making 45o with the x1 and x2 axes.
Solution 9 From balance of forces in x1 and x2 directions for section a-a
we have that
11 (A sin 60) 100A 200(A cos 60) sin 30 = 0
12 (A sin 60) 200(A cos 60) cos 30 = 0
which provides
11
12

= 100 3psi
= 100psi

46

In addition we have that

22 = 200 sin 60 = 100 3psi


The stress tensor is then specified by

100 3
100
0

= 100
100 3 0
0
0
0

For the surface on a 45o plane the unit normal is defined by n =( 12 , 12 , 0).
Then,

nn = ij ni nj = 11 n21 + 22 n22 + 2 12 n1 n2 = 100(1 + 3)psi

3.9

Equations of Motion in Cylindrical and Spherical Coordinates

Recall that equations of motion can be written as


div+b =u,tt

(3.37)

Namely,

ei kl ek el = kl,i ik el = lk,k el
xi
Therefore, equations of motion can be written as
=

+b =u,tt
Thus in curvilinear coordinates we need to evaluate .

3.9.1

Cylindrical Coordinate System (r, , z)

In that case we have

We may write then

b = (br , b , bz )
u = (ur , u , uz )

rr r
= r
zr z

rz
z
zz

= ij ei ej
= er ( rr er + r e + rz ez )
+e (r er + e + z ez )
+ez ( zr er + z e + zz ez )
47

(3.38)

or
= er tr +e t +ez tz
where
tr
t
tz

= rr er + r e + rz ez
= r er + e + z ez
= zr er + z e + zz ez

Recall that

+ e
+ ez ) (ur er + u e + uz ez )
r
r
z
ur
1 u
1
uz
+ ur +
+
r
r
r
z

u = (er
=

Therefore we can write that

+ e
+ ez ) (er tr +e t +ez tz )
r
r
z
tr
1 t
1
tz
+ tr +
+
r
r
r
z

= (er
=

which provides

rr
r
rz
er +
e +
ez
r
r
r
1
+ ( rr er + r e + rz ez )
r
1 r

z
+ (
er + r e +
e er +
ez )
r

zr
z
zz
+
er +
e +
ez
z
z
z

(3.39)

Based on Eqns. 3.38 and 3.39 we obtain the equations of motion in cylindrical
coordinates to be
rr
1 r
zr
1
+
+
+ ( rr ) + br
r
r
z
r
1
z
2
r
+
+
+ r + b
r
r
z
r
1 z
zz
1
rz
+
+
+ rz + bz
r
r
z
r

=
ur
=
u

(3.40)

=
uz

Similar procedure can be used in the spherical coordinate system.


Example 10 Invariants of the stress tensor. Let be a stress tensor in {xi }, i =
1, 2, 3

4 1 2
= 1 6 0
2 0 8

Find the invariants I , II , III .

48

Solution 10 The invariants are given by


I
II

III

= ii = 4 + 6 + 8 = 12
1
=
( kk ll kl lk )
2

11
11 12
+ det
= det
21 22
31

13
32

+ det

22
31

23
33

= 11 22 + 11 33 + 22 33 212 213 223 = 99


= det = 160

(3.41)

Now let us rotate x1 x2 axes about x3 axis for 45o counterclockwise. Then the
transformation matrix L becomes

2
2
0
2
2

2
L = 2
0
2
2
0
0 1

and thus in the new coordinate system the stress tensor assumes the values

6
1
2
0 = LLT = 1
2
4

2 2
8
The stress tensor invariants for 0 then are
I0
II0
III0

= 0ii = 18
1 0 0
=
( 0kl 0lk ) = 99
2 kk ll
= det 0 = 160

Thus the invariants remain the same in the two coordinate systems.

3.10

Mean and Deviator Stress Tensor

Experiments show that yielding and plastic deformation of many metals are
essentially independent of applied mean stress m defined by
m

=
=

1
( 11 + 22 + 33 )
3
1
( 1 + 2 + 3 )
3

(3.42)

Most plasticity theories postulate that the plastic behavior of materials is related
primarily to that part of the stress tensor which is independent of m . Therefore,
the stress tensor can be written as
= m + d
49

(3.43)

where
m
d

= diag[m , m , m ] = [ m ]
1
12
3 (2 11 22 33 )
1

(2

11 33 )
=
21
22
3
31
32

(3.44)

13

23
(3.45)
1
(2

)
33
11
22
3

where m is the mean stress tensor while d is the deviator stress tensor. The
latter measures deviation from the state of stress from a spherically symmetric
state (i.e., from the state of stress that exist in an ideal, frictionless fluid).
If the coordinate axes {xi }, i = 1, 2, 3, are the principal one, then 11 =
1 , 22 = 2 , 33 = 3 ,and ij = 0, for i 6= j. The invariants of m and d are
given by
Im
Id
IId
IIId

= 3 m ; IIm = 3 2m ; IIIm = 3m
= 0
1
= [( 1 2 )2 + ( 2 3 )2 + ( 1 3 )2 ]
6
1
=
(2 1 2 3 )(2 2 3 1 )(2 3 1 2 )
27

(3.46)

The eigenvalues of the deviator tensor d are (the principal stresses of d ) are
1
[(1 3 ) + ( 1 2 )]
3
1
= [(2 3 ) + ( 2 1 )]
3
1
= [(3 1 ) + ( 3 2 )]
3

S1

= 1 m =

S2

= 2 m

S3

= 3 m

(3.47)

Since S1 + S2 + S3 = 0, only two principal stresses of d are independent.

3.11

Generalized Hookes Law

Cauchy generalized Hookes law xx = Eexx into a statement that the components of stress are linearly related to the components of strain. Thus in tensor
notation
ij = Cijkl ekl
(3.48)
where Cijkl is a fourth order tensor of elastic constants of the material. For
isotropic materials C should be an isotropic tensor of order 4 or
Cijkl = ij kl + ik jl + il jk
Therefore,
ij = ekk ij + eij + eji

50

(3.49)

Since stress tensor is symmetric we must have = and consequently


ij = ekk ij + 2eij
The inverse relation is easily follows to be

eij =
ij
kk ij
2
3 + 2

(3.50)

(3.51)

Since E = 2(1 + ) and = (1 2)/2, we can write


eij =

1+

ij kk ij
E
E

(3.52)

In unabridged notation the last result becomes


exx

eyy

exx

exy

1
[ xx ( yy + zz )]
E
1
[ yy ( xx + zz )]
E
1
[ zz ( xx + yy )]
E
1
1
1
xy ; exz =
xz ; eyz =
yz
2
2
2

(3.53)

Frequently, Hookes law is expressed as


ij = 2(

ekk ij + eij )
1 2

(3.54)

Remark 6 Stresses are not unbounded as 1/2 (an incompressible material).


Generalized Hookes law ij = Cijkl ekl implies the following symmetry properties of tensor C due to symmetry of and e
Cijkl = Cjikl = Cijlk

(3.55)

In general, tensor C has 81 components. However due to symmetry conditions


expressed by Eq. 3.55, only 36 of these components are independent. This is
also clear from the fact that ij = Cijkl ekl is a set of six linear homogeneous
equations relating the six independent components of to six independent
components of e.
We next assume that there exits a strain energy function W (C) from which
the stress tensor can be derived through
ij =

W
eij

(3.56)

For small strains, W may be expressed in the power series


1
W = W0 + bij eij + Cijkl eij ekl +
2
51

(3.57)

where W0 , bij , ... are constants. Since the strain energy of the material has been
assumed to be zero in the undeformed state, W0 must be zero. Furthermore,
since e is symmetric we have that
W

1
1
= bij eij + Cijkl eij ekl + ... = bji eij + Cijkl eij ekl + ...
2
2
1
1
= bij eij + Cjikl eij ekl + ... = bij eij + Cijlk eij ekl + ...
2
2
1
= bij eij + Cklij eij ekl + ....
2

(3.58)

Therefore we conclude that


= bji
= Cjikl = Cijlk = Cklij

bij
Cijkl

(3.59)

Noting that
(Cijkl eij ekl )
= 2Cmnkl ekl
emn
we have that
ij =

W
= bij + Cijkl ekl + ...
eij

(3.60)

(3.61)

From Eq.(3.61) we conclude that the linear terms in the stress-strain relations
arise from the quadratic terms (in strains) of W. If the material is stress free in
the undeformed state, then bij = 0. It follows then that the linear constitutive
equation or the generalized Hookes law for an elastic solid is
ij
W

= Cijkl ekl
1
=
Cijkl eij ekl
2

(3.62)
(3.63)

Remark 7 The existence of the strain energy function introduces an additional


symmetry condition
(3.64)
Cijkl = Cklij
which reduces the number of independent constants of the tensor C from 36
to 21. Assuming the isotropic materials, the number of independent constants
reduces to two (, =Lam constants).
Isotropic materials have the strongest possible symmetry (i.e., a complete
symmetry about a point). There are some weaker symmetries naturally occurring (e.g., wood, crystals) or constructed (e.g., fiber-reinforced composites)
materials. For these materials the stress strain law must be form-invariant under certain transformations. Solids that do not have point symmetry are called
anisotropic.

52

3.12

Anisotropic Materials

Lets introduce the notation


11
e11

= 1 ; 22 = 2 ; 33 = 3 ; 23 = 4 ; 31 = 5 ; 12 = 6
= e1 ; e22 = e2 ; e33 = e; 2e23 = e4 ; 2e31 = e5 ; 2e12 = e6

(3.65)

Then the generalized Hookes law becomes


ij = cij ej ; i, j = 1, 2, 3, ..., 6

(3.66)

Obviously, the element cij are related to Cijkl and [cij ] is a 6-by-6 matrix. Since
Cijkl = Cklij, the so called stiness matrix [cij ] is symmetric, i.e.,
cij = cji
Therefore, there are 21 independent material constants.
With this new notation we have that
1
1
W =
cij ei ej = i ei
2
2
W
i =
ei

(3.67)

(3.68)

In expanded from the strain energy function becomes


1
c11 e21 + c12 e1 e2 + c13 e1 e3 + c14 e1 e4 + c15 e1 e5 + c16 e1 e6
2
1
+ c22 e22 + c23 e2 e3 + c24 e2 e4 + c25 e2 e5 + c26 e2 e6
2
1
+ c33 e23 + c34 e3 e4 + c35 e3 e5 + c36 e3 e6
2
1
+ c44 e24 + c45 e4 e5 + c46 e4 e6
2
1
+ c55 e25 + c56 e5 e6
(3.69)
2
Most solids exhibit symmetry properties with respect to certain rotations
of the body or reflection about one or more planes. These properties further
reduce the number of elastic constants. Obviously, the elastic constants cij , in
general, depend upon the reference frame, since the stress components i and the
strain components ei vary with dierent coordinate systems. For certain solids
the elastic constants cij may remain invariant under a given transformation of
coordinates. It is this invariance property that determines the elastic symmetry
of the solid.
W

3.12.1

Material With One Plane of Symmetry

Say the plane of symmetry is the x1 x2 plane. Then cij must be invariant under
the transformation
x01 = x1 ; x02 = x2 ; x03 = x3
(3.70)
53

which defines the transformation matrix

1 0 0
L = 0 1 0
0 0 1

(3.71)

and the strain tensor

e11
e0 = LeLT =

e12
e22

e13
e23
e33

(3.72)

where denote the symmetric part of the matrix. It follows than from Eq.
(3.72) that
e023 = e23 ; e031 = e31
with all the other components unchanged. Thus
e04 = e4 ; e05 = e5 ; e0i = ei ; i = 1, 2, 3, 6

(3.73)

The strain energy function W 0 is obtained from the right hand side of Eq.
3.69 by changing the signs of e4 and e5 . Since the material is symmetric about
the x1 x2 plane, we must have that
W0 = W

(3.74)

c14 = c15 = c24 = c25 = c34 = c46 = c56 = 0

(3.75)

Consequently, we have that

Therefore, we are left with 13 (=21-8) independent elastic constants. Consequently, for an elastic material with x1 x2 plane of symmetry, the generalized
Hookes law i = cij ej can be written as

e1
c11 c12 c13 0
0 c16
1

2 c12 c22 c23 0


0 c26

e2

3 c13 c23 c33 0


0
c
e
36 3

(3.76)

4 = 0
0
0
c
c
0
e
44
45

4

5 0
0
0 c45 c55 0 e5
c16 c26 c36 0
6
0 c66
e6

3.12.2

Orthotropic Materials

Definition 15 If a material has three mutually orthogonal planes of symmetry,


it is called orthotropic.
Wood is an example of such material. Say that the planes of symmetry are
the coordinate planes. Then, the symmetry about the x1 x2 plane implies that
(see Eq. 3.75)
c14 = c24 = c34 = c56 = c15 = c25 = c35 = c46 = 0
54

Similarly, the symmetry about the x1 x3 plane requires that


c16 = c26 = c36 = c45 = c15 = c25 = c35 = c46 = 0

(3.77)

Comparison of Eqns. 3.75 and 3.77 shows that Eq. 3.77 requires only four additional constants to vanish. Thus the number of independent constants reduces
to 9 (=13-4). Finally for material with symmetry about the x1 x3 plane implies
that
e04 = e4 ; e06 = e6
and thus
c14 = c16 = c24 = c26 = c34 = c36 = c45 = c56 = 0

(3.78)

which bring no new conditions. Therefore, if x1 x2 and x2 x3 planes are planes


of symmetry, then x1 x3 plane is also a plane of symmetry. The Hookes law
for orthotropic material is then

e1
1
c11 c12 c13 0
0
0

2 c12 c22 c23 0

0
0


e2
3 c13 c23 c33 0
e3
0
0

(3.79)

4 = 0

0
0 c44 0
0


e4
5 0
0
0
0 c55 0 e5
0
0
0
0
0 c66
6
e6
It is obvious from Eq. 3.79 that e4 = e5 = e6 = 0 implies that 4 = 5 =
6 = 0, which means that the principal directions of stress coincide with the
principal directions of strain. This is not so for monoclinic materials (see Eq.
3.76) where e4 = e5 = e6 = 0, implies that 6 = c16 e1 + c26 e2 + c36 e3 which
may not vanish.

3.12.3

Transversely Isotropic Materials

If an orthotropic solid exhibits symmetry with respect to arbitrary rotations


about one of the axes (such as the x3 -axis), then it is called transversely
isotropic. Say that the system {x0i } is obtained from the system {xi } by a
rotation about the x3 -axis for an angle . Then x0 = Lx, where

cos sin 0
L = sin cos 0
0
0
1

Then cij must be invariant under the transformation L. Since


e0ij = lir ljs ers

55

we can calculate that


e011
e022
e033
e023
e031
e012

=
=
=
=
=
=

e11 cos2 + e22 sin2 + 2e12 sin cos


e11 sin2 + e22 cos2 2e12 sin cos
e33
e23 cos e31 sin
e23 sin + e31 cos
(e11 e22 ) sin cos + e12 (cos2 sin2 )

e01
e02
e03
e04
e05
e06

=
=
=
=
=
=

e1 cos2 + e2 sin2 + e6 sin cos


e1 sin2 + e2 cos2 e6 sin cos
e3
e4 cos e5 sin
e4 sin + e5 cos
2(e1 e2 ) sin cos + e6 (cos2 sin2 )

or

(3.80)

The strain energy function W 0 is obtained by replacing e0i with ei in Eq. 3.69. If
the solid is symmetric about an arbitrary rotation about the x3 axis we must
have W 0 = W for an arbitrary . By equating the coecient of e21 we get
c11 = c11 cos4 + c22 sin4 + 2(c12 + 2c66 ) sin2 cos2
which can be rearranged to
c11 (1 + cos2 ) = c22 (1 cos2 ) + 2(c12 + 2c66 ) cos2

(3.81)

for all . Therefore, we must have


c11 = c22 ; c11 = c22 + 2(c12 + 2c66 )

(3.82)

which implies that


1
(3.83)
(c11 c12 )
2
By equating the coecients of e1 e3 and e24 in W 0 = W it follows that
c66 =

c13 e1 e3

= c13 e01 e03 + c23 e02 e03


= c13 (e1 cos2 + e2 sin2 + e6 sin cos )e3
+c23 (e1 sin2 + e2 cos2 e6 sin cos )e3

and therefore
c13
1
c44
2

= c13 cos2 + c23 sin2


1
1
=
c44 cos2 + c55 sin2
2
2
56

which implies that


c13
c44

= c23
= c55

Thus for transversely isotropic material we


elastic constants. The Hookes law becomes

1
c11 c12 c13 0
2
c12 c11 c23 0

= c13 c23 c33 0


4
0
0
0 c44

5
0
0
0
0
0
0
0
0
6
1
c66 =
(c11 c12 )
2

3.12.4

(3.84)
(3.85)
need only 9-4=5 independent
0
0
0
0
c44
0

0
0
0
0
0
c66

e1
e2
e3
e4
e5
e6

(3.86)

Isotropic Materials

Here, the elastic constants cij are independent of the orientation of the coordinate axes. We start with the stress-strain relationship for an orthotropic
material. The symmetry with respect to rotation about the x3 axis implies
that
1
c11 = c22 ; c66 = (c11 c22 )
2
(3.87)
c13 = c23 ; c44 = c55
Similarly, the symmetry with respect to rotation about the x1 axis requires
that
1
c22 = c33 ; c44 = (c22 c33 )
2
(3.88)
c12 = c13 ; c55 = c66
By combining Eqns. 3.87 and 3.88 results in
c11
c12
c44

= c22 = c33
= c13 = c23 =
1
= c55 = c66 = (c11 c12 ) =
2

which gives
c11 = c22 = c33 = + 2
Therefore, Hookes law for an isotropic solid is given


1
+ 2

0 0
2

+
2

0
0

+
2
0
0


4 =
0
0
0


5
0
0
0
0
0
0
0
0 0
6
57

(3.89)
by
0
0
0
0
0

e1
e2
e3
e4
e5
e6

(3.90)

Based on this result we have


1
2
3
4
5
6

=
=
=
=
=
=

(e1 + e1 + e1 ) + 2e1
(e1 + e1 + e1 ) + 2e2
(e1 + e1 + e1 ) + 2e3
e4
e5
e6

or
ij = ekk ij + 2eij

(3.91)

This result can be written as


ij
Cijkl

= Cijkl ekl
= ij kl + ( ik jl + il jk )

(3.92)
(3.93)

The strain energy function then becomes


W

=
=

3.13

1
1
ij eij = (ekk )2 + eij eij
2
2
1
(ekk )2 + (e21 + e22 + e23 + 2e223 + 2e213 + 2e212 )
2

(3.94)

Infinitesimal Strains in Cylindrical and Spherical Coordinates

Recall that the elements of strain tensor are defined by


eij =

1
(ui,j + uj,i )
2

Then, the strain dyadic is given by


e = eij ei ej =

1
(uj,i ei ej + ui,j ei ej )
2

(3.95)

Since
uj,i ei ej
ui,j ei ej

(uj ej ) = u
xi

T
= (ui,j ej ei )T = [ej
(ui ei )]T = (u)
xj
= ei

we have that

(3.96)

1
T
(3.97)
(u+(u) )
2
This equation can be used to obtain the strain-displacement relations in various
curvilinear coordinate systems.
e=

58

3.13.1

Cylindrical Coordinates (r, , z)

The displacement vector is then written as


u =ur er + u e + uz ez
and the components of the infinitesimal strain dyadic e are given by

err er erz
er e ez
ezr ez ezz

(3.98)

By recalling that

u = (er

+ e
+ ez )(ur er + u e + uz ez )
r
r
z

we have
u =

1
u
uz
ur
er er + (ur +
)e e +
ez ez
r
r

z
1 uz
u
ur
uz
+
e ez +
ez e +
ez er +
er ez
r
z
z
r
u
1 ur
+
er e + (
u )e er
r
r

(3.99)

By noting, for example, that ez = e e ez , Eqns. 3.97 and 3.99 lead to the
following results

3.13.2

ur
;
r

err

ez

= ez =

ezr

= erz

er

= er

e =

1 u
(
2 z
1 ur
= (
2 z
1 ur
= (
2 r

1 u
(
+ ur );
r
1 uz
+
)
r
uz
+
)
r
u
u
+
)
r
r

ezz =

uz
z

(3.100)

Spherical Coordinates (R, , )

Now the displacement vector is given by


uR eR + u e + u e
and the components of the infinitesimal strain dyadic are given by

eRR eR eR
eR e e
eR e e
59

(3.101)

Since
1
u
1
u
uR
eR eR + (uR +
)e e +
(sin uR + cos u +
)e e
R
R

R sin

1 u
1
1
u
uR
+
e e +
(
cos u )e e +
(
sin u )e eR
R
R sin
R sin
u
u
1 uR
+
(3.102)
eR e +
eR e + (
u )e eR
R
R
R

u =

and by using Eq. 3.97 we get


eRR
e
eR
eR

uR
1 u
1
u
; e = (
+ uR ); e =
[
+ sin uR + cos u ]
R
R
R sin
1
1 u
u
= e =
(
+
cot u )
2R sin

1
1 uR u u
= eR = (
+

)
2 R sin
R
R
1 1 uR
u
u
= eR = (
+
)
(3.103)
2 R
R
R
=

This procedure can be extended to other curvilinear coordinate systems.

3.14

Constitutive Equations in Cylindrical and


Spherical Coordinates

In dyadic notation, we can write generalized Hokkes law for isotropic materials
as
= dI + 2e
(3.104)
where , I, and e are stress, unit, and strain dyadics, respectively and
d = ekk = u

(3.105)

Using Eq. 3.97 then produces


T

= dI + (u+(u) )

(3.106)

ij ei ej = d ij ei ej + 2eij ei ej

(3.107)

or

60

Then by comparing the coecients of the like terms ei ej of Eq. 3.107produces


in cylindrical coordinates (r, , z)
ur
r
2 u
d + 2ezz = d +
(
+ ur )
r
uz
d + 2ezz = d +
z
ur
uz
rz = 2ezr = (
+
)
z
r
u
1 uz
z = 2ez = (
+
)
z
r
1 ur
u
u
r = 2er = (
+
)
r
r
r

rr

= d + 2err = d + 2

zz

zr

(3.108)

where

ur
1 u
uz
ur
+
+
+
(3.109)
r
r
r
z
Similar approach in spherical coordinate system results in
uR
RR = d + 2eRR = d + 2
R
2 u
= d + 2e = d +
(
+ uR )
R
2 u
= d + 2e = d +
(
+ sin uR cos u )
R sin
1 u
u
= 2e = (
+
cot u )
R sin

1 uR
u
u
R = 2eR = (
+

)
R sin
R
R
1 uR
u
u
R = 2eR = (
+
)
(3.110)
R
R
R
where
1 u
uR 2uR 1 u
+
(
+ cot u ) +
(3.111)
d=u=
R
R R
R sin
d=u=

3.15

Dilatation d

Let us consider the physical interpretation of d = ekk . For that purpose lets
consider a cube of size 1 2 3 before deformation. Let e11 , e22 , and e33 be
the principal strains, i.e., eij = 0 for i 6= j. Then, after the deformation we have
that
1
2
3

+
2+
3+
1

61

1 e11
2 e22
3 e33

Remark 8 The fibers of the cube will remain perpendicular to each other.
The initial and final volumes of the cube are given by
V0
V

= 1 2 3
= (1 + e11 )(1 + e22 )(1 + e33 )V0

Then the change in volume is


V V0 = V0 (e11 + e22 + e33 + ...)
and by neglecting the higher order terms (infinitesimal strain theory) we obtain
the relative change of volume to be
V V0
= ekk = d
V0

(3.112)

Thus the dilatation represents the relative change of volume of the material
during the deformation.

62

Chapter 4

Boundary Value Problems


of Linear Elasticity
Consider an elastic solid of volume V and boundary B = Bu B (see Fig. 4.1)
where Bu and B denote the portions of the boundary where displacement or
stress field is being prescribed, respectively.
Then the equilibrium equations are given by
ij,j + fi = 0; x V

(4.1)

where f denotes body force per unit volume. Constitutive equations are given
by
ij = Cijkl ekl
(4.2)
where the tensor of elastic properties satisfies the following symmetry relations
Cijkl = Cjikl = Cijlk = Cklij

(4.3)

Strain-displacement relations are given by


eij =

1
(ui,j + uj,i )
2

(4.4)

while the compatibility conditions are prescribed by


eij,kl + ekl,ij eik,jl ejl,ik = 0

(4.5)

The boundary conditions are specified in the following form


ui
ij nj

= ui (B); x Bu
= Tin (B); x B

(4.6)
(4.7)

Definition 16 If only displacements are prescribed on the boundary B, Eqns.


4.1 to 4.7 define the displacement boundary value problem.
63

Bs
V
/V=B

Bu

Figure 4.1: Boundary value problem of elasticity.


Definition 17 If only tractions are prescribed on the boundary B, Eqns. 4.1
to 4.7 define the stress boundary value problem,
Definition 18 If both displacements and tractions are prescribed on the boundary B, Eqns. 4.1 to 4.7 define the mixed boundary value problem.
At an in interface C between two dierent materials, certain continuity conditions must be enforced. For a perfect bond both displacement and traction
fields must be continuous across the interface, i.e.,
(1)

ui

n(1)
Ti

(2)

= ui ; x C

(4.8)

(4.9)

n(2)
Ti ; x

For isotropic materials we have that


Cijkl = ij kl + ( ik jl + il jk )
which results in the constitutive equations of the form
ij = ekk ij + 2eij
Remark 9 Note that
2
kk = (3 + 2)ekk = 3( + )ekk
3
64

(4.10)

or
s = 3d

(4.11)

where
s = kk
d = ekk
2
= +
3

(4.12)

with being the bulk modulus.


Remark 10 The inverse form of the constitutive equation 4.10 is given by
eij =

1+

ij s ij
E
E

(4.13)

In cylindrical coordinates (r, , z) the equations of equilibrium are


rr
1 r
zr
1
+
+
+ ( rr ) + br
r
r
z
r
1
z
2
r
+
+
+ r + b
r
r
z
r
1 z
zz
1
rz
+
+
+ rz + bz
r
r
z
r

= 0
= 0

(4.14)

= 0

The stress-strain relationships are


rr
zr

= d + 2err ; = d + 2ezz ; zz = d + 2ezz


= rz = 2ezr ; z = z = 2ez ; r = r = 2er

(4.15)

where d = u = err + e + ezz .


The strain displacement equations are given by Eq. 3.100.
In spherical coordinates (R, , ) the equations of equilibrium are given by
RR
1 R
1 R
1
+
+
+ (2 RR + R cot ) + fR = 0
R
R
R sin
R
1
1
1
R
= 0
+
+
+ [( ) cot + 3 r ] + f (4.16)
R
R
R sin
R
R
1
1
1
+
+
+ (2 cot + 3 R ) + f = 0
R
R
R sin
R
The stress-strain and strain-displacement relations are given by Eq. 3.110.

4.1

Naviers Equations

Equations of equilibrium are given by Eq. 4.1. Then by using constitutive


equations for an isotropic solid (Eq. 4.10) together with definition of strain the
equilibrium equations become
( + )uj,ji + ui,jj + fi = 0
65

(4.17)

or
( + )( u)+2 u + f = 0

(4.18)

These equations are known as Naviers equations. In cylindrical and spherical


coordinates we can utilize the expressions for f, u, and 2 f in order to
obtain Naviers equations for these curvilinear coordinates.

4.2

Uniqueness of Solution

Theorem 9 Let the tractions be prescribed over a part B of the boundary


B = V and displacements be prescribed over the remaining part of Bu (note
B Bu = B). Then the solution of elastostatic problem is unique. r = r =
2er
Proof. Assume that two solutions u(1) and u(2) are possible. Then we have
that
(1)
(2)
ij,j + fi = 0; ij,j + fi = 0
where

(1)(2)

ij

(1)(2)

= Cijkl uk,m

The boundary conditions are given by


(1)

ui

(1)
ij nj

(2)

= ui
=

= ui (B) ; x Bu

(2)
ij nj

= ti (B); x B

Let us define
ui
ij

(1)

(2)

ui ui
= Cijkm uk,m

Then it is easy to show that


ij,j
ui
ij nj

= 0; x V
= 0; x Bu
= 0; x B

Let W be the strain energy function corresponding to the displacement ui . Then


Z
Z
Z
W dV =
ij eij dV =
ij ui,j dV
2
V
V
Z
ZV
( ij ui ),j dV
ij,j ui dV
=
V
V
Z
Z
ij ui nj dB
ij,j ui dV = 0
=
B

since ij,j = 0; x V and ij nj = 0; x B while ui = 0; x Bu .


66

R
Since W is nonnegative, the fact that V W dV = 0, implies that W = 0 in
V. Since W is positive definite quadratic form in strain components, it cannot
be zero unless all the strain components are zero. Thus eij = 0 implies that
(1)
(2)
ui ui ui represents a rigid body motion. Since ui = 0 on Bu we must
(1)
(2)
have that ui = 0 everywhere in the body and thus we have that ui = ui .
Remark 11 This proof applies to the linear case only.
Theorem 10 (Clapeyrons) If a solid is in equilibrium under the action of a
given system of body and surface forces, then the strain energy of deformation
is equal to one-half the static work that would be done by the external forces.
Proof. From the uniqueness proof we have that
Z
Z
Z
2
W dV =
ij eij dV =
ij ui,j dV
V
V
V
Z
Z
( ij ui ),j dV
ij,j ui dV
=
ZV
ZV
ij ui nj dB +
fi ui dV
=
Z V
ZB
Tin ui dB +
fi ui dV
=
B
V
Z
Z
Tn udB +
f udV
=
B

which completes the proof.


Theorem 11 (Betti-Rayleigh Reciprocity (BRR) Theorem)If an elastic body is
subject to two systems of body and surface forces, then the work done by the first
system of forces acting through the displacements of the second system is equal
to work done by the second system of forces acting through the displacements of
the first system.
Proof. Let us consider two equilibrium states: u(1) due to body forces
f and tractions t(1) and u(2) due to f (2) and t(2) . Then the work done by
the forces f (1) and tractions t(1) if they acted through the displacement u(2) is
Z
Z
(1) (2)
(1) (2)
=
fi ui dV +
ti ui dB
V
B
Z
Z
(1) (2)
(1)
(2)
fi ui dV +
ij nj ui dB
=
ZB
ZV
(1) (2)
(1) (2)
fi ui dV + (ij ui ),j dV
=
V
Z
ZV
(1)
(2)
(1) (2)
(1)
(f + ij,j )ui dV +
ij ui,j dV
=
V
V
Z
(1) (2)
ij ui,j dV
(4.19)
=
(1)

67

Thus

1
=
2

(1) (2)
ij (ui,j

(2)
uj,i )dV

(1) (2)

Cijkl ekl eij dV

(1) (2)

(4.20)
(1)

Since Cijkl = Cklij , then the integrand Cijkl ekl eij is symmetric in ekl and
(2)

eij . Thus we have that


Z
(2) (1)
(2) (1)
Cijkl ekl eij dV =
ij ui,j dV
V
V
Z
Z
(2) (1)
(2) (1)
fi ui dV +
ti ui dB
=
Z

By comparing Eqns. 4.19 and 4.21 we obtain that


Z
Z
Z
Z
(1) (2)
(1) (2)
(2) (1)
(2) (1)
fi ui dV +
ti ui dB =
fi ui dV +
ti ui dB
V

or equivalently

(1) (2)
ij ui,j dV

(4.21)

which completes the proof.

(4.22)

(2) (1)

ij ui,j dV

(4.23)

Example 11 Using Betty-Rayleigh resiprocity theorem derive the change of volume for an elastic body subjected to a body force and surface tractions.
(1)

Solution 11 Let ui = Axi be the solution of a problem of an isotropic solid


(1)
of arbitrary volume with no body forces subjected to the traction ti = 3Ani ,
where = + 2/3 bulk modulus. Then it follows that
(1)

eij = A ij ij = 3A ij
Let ui , eij , fi and ti denote a second possible state of the body. Then by putting
(1)
fi = 0 and using the Betty-Rayleigh reciprocity theorem we obtain
Z
Z
Z
(1) (2)
(2) (1)
(2) (1)
ij eij dV =
fi ui dV +
ti ui dB
V

By substituting

(2)
eij

=
Z

But

(2)
eij , fi

(2)
ti

= fi and
= ti we get
Z
Z
3eii dV =
fi xi dV +
ti xi dB
V

eii dV = V increase in volume due to deformation, thus


Z
Z
1
ti xi dS}
V =
{ fi xi dV +
3 V
S

which represents increase in volume of an elastic body due to the body force fi
and the surface traction ti .
68

4.3

Exact Solutions for Some Problems

For some problems in elastostatics, Naviers equations can be integrated directly.


The integration constants can be found by using the boundary conditions.
Example 12 Consider a cylindrical tube of inner and outer radii a and b, respectively subjected to internal pressure p and the external pressure q. Solve for
the unknown displacement field in the tube.
Solution 12 Assume that there is no displacement in the z direction, i.e.
uz 0. Due to symmetry we have
ur = u(r); u = uz = 0
The boundary conditions are then given by
rr (r = a) = p; rr (r = b) = q
Recall that the equations of equilibrium are given by (no body forces)
( + )( u) + 2 u = 0
Furthermore, in cylindrical coordinates (r, , z) we have that

+ e
+ ez )f
r
r
z
1
ur
ur 1 u
uz
u =
+
+
= u,r + u
r
r r
z
r
u
( u) = er (u,r + ),r
r
f

= (er

We still need the term 2 u in cylindrical coordinates. In general we have that


2 u = u = (er

+ e
+ ez ) (u)
r
r
z

However
u =

1
u
uz
ur
er er + (ur +
)e e +
ez ez
r
r

z
1 uz
u
ur
uz
+
e ez +
ez e +
ez er +
er ez
r
z
z
r
u
1 ur
+
er e + (
u )e er
r
r

therefore,
1

u = ur,rr er er + [ (ur + u, )],r e e + uz,zr ez ez


r
r
1
+( uz, ),r e ez + u,zr ez e + ur,zr ez er + uz,rr er ez
r
1
+u,rr er e + [ (ur, u )],r e er
r
69

Consequently
er

(u) =ur,rr er + uz,rr ez + u e


r

where we have used the fact that

r er

r e

r ez

= 0. Similarly we get

(u) = uz,zz ez + u,zz e + ur,zz er


z
1
ur
2
1
1
e
(u) = ( ur,r 2 2 u + 2 ur, )er
r
r
r
r
r
2
1
1
1
+( 2 ur, + u,r 2 u + 2 u, )e
r
r
r
r
1
1
+( 2 uz, + uz,r )ez
r
r
ez

By collecting the last three results we obtain


1
ur
2
1
2 u = (ur,rr + ur,r 2 2 u, + 2 ur, + ur,zz )er
r
r
r
r
1
1
1
2
( 2 u, + u,r 2 u + 2 ur, + u,rr + u,zz )e
r
r
r
r
1
1
+(uz,zz + uz,r + 2 uz, + uz,rr )ez
r
r
For axisymmetric case u =ur er = u(r)er , and / = /z 0 and we get
1
ur
1
2 u = (ur,rr + ur,r 2 )er = (u,r + u),r er
r
r
r
Therefore, the equation of equilibrium for the cylinder problem becomes
( + 2)

d du 1
(
+ u) = 0
dr dr
r

which can be solved for displacement


u = Ar +

B
r

Corresponding stress field follows from the equation rr = d + 2err , where


du u
+ = 2A
dr
r
du
B
=A 2
dr
r

d = u=
err

Thus
rr = 2( + )A 2

70

B
r2

Similarly

= d + 2e = 2( + )A + 2

zz
rr +

B
r2

u
B
=A+ 2
r
r
= d = (rr + )
rr +
= 2( + )d d =
2( + )

Then the boundary conditions imply


B
= p
a2
B
rr (b) = q 2( + )A 2 2 = q
b

rr (a) = p 2( + )A 2

which can be solved for


A=

pa2 qb2
a2 b2 (p q)
;B =
2
2
2( + )(b a )
2(b2 a2 )

Thus the displacement and stress fields are given by


B
r
pa2 qb2
=

b2 a2
pa2 qb2
=
+
b2 a2
pa2 qb2
= 2 2
b a2

u = Ar +
rr

zz

a2 b2 (p q)
(b2 a2 )r2
a2 b2 (p q)
(b2 a2 )r2

If there is no external pressure (q = 0) we have


rr

pa2
(1
a2
pa2
(1 +
b2 a2
b2

b2
) compression
r2
b2
) tension
r2

The maximum (minimum) stresses are then


max
min

a2 + b2
>p
b2 a2
2a2 p
= (r = b) = 2
< min
b a2
= (r = a) = p

For thin cylinders d = b a << 1 and a, b and r are nearly equal. Thus we have
=

pa2
b2
pa2
pa
b2
(1
+
)
=
)
(1
+
b2 a2
r2
d(b + a)
r2
d
71

which implies that


b >> a which implies that
A =
B

1
d.

For thick pipes on the other hand we have that

pa2 qb2
q

2
2
2( + )(b a )
2( + )
a2 (p q)
a2 b2 (p q)

2(b2 a2 )
2

and thus
q
p q a2
r+
2( + )
2 r
2
a
= q (p q) 2
r
a2
= q + (p q) 2
r

u =
rr

For p = 0 we get that (r = a) = 2q, i.e. twice the stress without the hole.
Finally, for r >> a we have that
qr
2( + )
= = q

u =
rr

Example 13 Determine the stress in a rotating shaft ( = const. and no longitudinal deformation).
Solution 13 Due to symmetry, ur = u(r), and u = uz = 0. The Naviers
equation becomes
d du 1
( + 2) (
+ u) + 2 r = 0
dr dr
r
Let
2
C
+ 2
then the equation of equilibrium becomes
d du 1
(
+ u) = Cr
dr dr
r
which can be solved for
u = Ar +

r3
B
C
r
8

72

For a bounded solution at r = 0 we choose B = 0. Thus


r3
8
du u
1
d = u=
+ = 2A Cr2
dr
r
2
du
C
rr = d + 2err = d + 2
= 2( + )A (2 + 3)r2
dr
4
u
C
= d + 2e = d + 2 = 2( + )A (2 + )r2
r
4
C 2
zz = d = 2A r
2
u = Ar C

Now the boundary condition at the surface of the shaft is given by


rr (r

= a) = 0 2( + )A

A =

C
(2 + 3)a2 = 0
4

2 + 3
Ca2
8( + )

Consequently, the displacement and the stress fields are given by


u =
rr

zz

Cr 2 + 3 2
(
a r2 )
8 +
C
(2 + 3)(a2 r2 )
4
C
2 + 3 2
(2 + )(
a r2 )
4
2 +
C 2 + 3 2
(
a r2 )
2 2 +

The the displacement and stress fields on the surface of the shaft are
u(r
(r
zz (r

Ca3
( + 2)
8( + )
Ca2
= a) =
2

2
= a) = a
4( + )
= a) =

Maximum stress at r = 0 is then


rr (0) = (0) =

1 2
Ca (2 + 3)
4

Example 14 Consider a thin plate in x1 x2 plate with an elastic circular inclusion loaded as shown in Fig. 4.3. Determine the unknown displacement and
stress fields in the plate and the inclusion.
73

s
0

r
s
0

l,m
2

l,m

a
2

Figure 4.2: An elastic plate with a circular inclusion loaded at infinity.

74

Solution 14 Due to axisymmetry u =u(r)er and the solution of the Naviers


equation becomes
Bi
u(i) = Ai r +
; i = 1, 2
r
so that the stress field is given by
(i)
rr
(i)

(i)
zz

2i Bi
r2
2i Bi
= 2(i + i )Ai +
r2
(i)
(i)
= i ( rr + ) = 4 i (i + i )Ai
= 2(i + i )Ai

where Ai , Bi ; i = 1, 2 are the unknowns. Requirement of finite displacement at


(1)
r = 0 requires that B = 0. In addition as r , we must have that rr 0
or that
0
2(1 + 1 )A1 = 0 A1 =
2(1 + 1 )
Continuity of displacement and traction fields at r = a then is expressed as
u(1) (a) = u(2) (a)
(2)
(1)
(2)
t(1)
r (a) = tr (a) rr (a) = rr (a)
which gives
A1 a +
2(1 + 1 )A1

B1
a

21
B1
a2

= A2 a
= 2(2 + 2 )A2

The last two equations can be solved for the unknowns


A2
B1

0 (1 + 21 )
(1 + 1 )(1 + 2 + 2 )
1 + 1 2 2
= 0 a2
(1 + 1 )(1 + 2 + 2 )
=

Therefore, we can evaluate both displacement and stress fields throughout the
medium. In particular, the interface stresses between the plate and the inclusion
are calculated to be
(2)
(1)
rr (a) = rr (a) = 2(2 + 2 )A2
2 B1
(1)
(a) = 2(1 + 1 )A1 + 12
a
(i)
zz
= 4 i (i + i )Ai ; i = 1, 2

Therefore, and zz have discontinuity at the interface r = a.

75

Example 15 Consider a spherical shell a R b subjected to internal pressure p and external pressure q. Thus the boundary conditions are specified by
RR (R = a) = p
RR (R = b) = q
Due to point symmetry uR = u(R); u = u 0. For the Naviers equations (no
body forces)
( + )( u) + 2 u = 0
we have for spherical coordinates with point symmetry that
u=

du
d du
2u
2u
+
( u) = eR
(
+
)
dR
R
dR dR
R

We still need the 2 uterm. Thus we consider

1
2 u = u = eR
+ e
+ e
(u)
R
R
R sin
But
u =u,R eR eR +

u
u
e e + e e
R
R

and thus

d du
(u) =
eR
R
dR dR

1 du
u
e
)eR
(u) = (

R
R dR R2

1
1 du
u
e
)eR
(u) = (

R sin
R dR R2
eR

Consequently we obtain
d2 u
1 du
u
+ 2(
2 )]
2
dR
R dR R
d du
u
(
+2 )
dR dR
R

2 u = [
=

Therefore, the Naviers equations reduce to


d du
u
(
+2 )=0
dR dR
R
Consequently, it follows that the displacement field is given by
u = AR +

76

B
R2

and the corresponding stress filed is


RR

= d + 2eRR = u + 2

du
dR

du
u
du
+ 2 ) + 2
dR
R
dR
B
(3 + 2)A 4 3
R
du
u
u
d + 2e = (
+ 2 ) + 2
dR
R
R
B
(3 + 2)A + 2 3
R
du
u
u
d + 2e = (
+ 2 ) + 2
dR
R
R
B
(3 + 2)A + 2 3
R

= (
=

=
=

=
=

Then from the boundary conditions we have that


B
a3
B
(3 + 2)A 4 3
b

(3 + 2)A 4

= p
= q

from which it follows that


A =
B

pa3 qb3
(3 + 2)(b3 a3 )
(p q)a3 b3
4(b3 a3 )

Corresponding displacement and stress fields are then


(p q)a3 b3 1
pa3 qb3
R+
3
3
(3 + 2)(b a )
4(b3 a3 ) R2
pa3 qb3 (p q)a3 b3 1
=

(b3 a3 )
(b3 a3 ) R3
pa3 qb3 (p q)a3 b3 1
= = 3
+
(b a3 )
2(b3 a3 ) R3

u =
RR

If q = 0 we obtain

pa3
4
b3
(
R + 2)
3
3
4(b a ) 3 + 2
R
3
3
pa
b
= 3
(
1) 0
b a3 R3
pa3
b3
= = 3
(1
+
)>0
b a3
2R3

u =
RR

77

(4.24)
(4.25)
(4.26)

The maximum stresses at R = a are


pa3
b3
(1
+
)
b3 a3
2a3
For a thin shell, d = b a << a it follows that
RRMax = Max =

= =

pa3
pa
b3
)
(1
+
2
2
3
(a + ab + b )d
2R
2d

For a very thick sphere, b >> a, we have that


(p q)a3
qR
+
3 + 2
4R2
3
a
RR q (p q) 3
R
a3
= q + (p q) 3
2R
Consequently at R = a we have that
u =

1
(p 3q)
2
Thus for a thick sphere and no internal pressure (p = 0) we have that
RR = p; = =

3
= q
2
demonstrating that there is a stress concentration taking place at the cavity.

4.4

Special Models

4.4.1

1-D Models

Definition 19 If the body force and the components of the stress tensor depend
upon one spatial variable, say x1 , we talk about 1-D models.
Equation of equilibrium then becomes
i1,1 + fi = 0

(4.27)

and we distinguish three cases:


1. Longitudinal Strain
u1 = u1 (x1 ); u2 = u3 = 0
Then the strain and stress tensors are given by

u1,1 0 0
0 0
e = 0
0
0 0

( + 2)u1,1
0
0
0
u1,1
0
=
0
0
u1,1
78

(4.28)

(4.29)

Equation of equilibrium is given by


( + 2)u1,11 + fi = 0

(4.30)

11 = 11 (x1 ); ij = 0; i, j 6= 1

(4.31)

2. Longitudinal Stress

Thus the stress tensor is defined by

11
= 0
0

Since

22
33

0 0
0 0
0 0

(4.32)

= ekk + 2e22 = 0
= ekk + 2e33 = 0

it is easy to show that


e22 = e33

(4.33)

The from the condition 22 = 0 it follows that


e22 = e33 =

e11 = e11
2 ( + )

(4.34)

where denotes the Poissons ratio. Finally, the stress filed is described
by
3 + 2
(4.35)
11 =
e11 = Ee11
+
where E denotes Youngs modulus.
3. Shear, defined in terms of displacement field which is in the plane perpendicular to the x1 axis
u =u2 (x1 )e2 + u3 (x1 )e3
Then the strain and stress tensors are given by

1
1
0
2 u2,1
2 u3,1
1
0
0
e = 2 u2,1
1
u
0
0
3,1
2
0
u2,1 u3,1
0
0
= u2,1
u3,1
0
0

(4.36)

(4.37)

The Naviers equation reduce to two uncoupled equations


u2,11 + f2
u3,11 + f3
79

= 0
= 0

(4.38)

4.4.2

Two-dimensional Problems

Definition 20 The body forces and stresses are independent of one spatial coordinate, say x3 .
Therefore,

x3

0 and the Naviers equations reduces to


3, + f3
, + f

= 0
= 0; , = 1, 2

(4.39)
(4.40)

or
31,1 + 32,2 + f3
11,1 + 12,2 + f1
21,1 + 22,2 + f2

= 0
= 0
= 0

(4.41)
(4.42)
(4.43)

It can be seen that Eqn. 4.41 is decoupled from the Eqns. 4.42 and 4.43.
Antiplane Strain Model
Definition 21 In this model the displacement field is assumed to be of the form
u3 = u3 (x1 , x2 ); ui = 0; i = 1, 2

(4.44)

Physically, this corresponds to an infinitely long cylinder (along the x3 axis


loaded uniformly with tractions in the xdirection. Then the corresponding
strain and stress tensors are

1
0
0
2 u3,1
1

0
e = 0
(4.45)
2 u3,2
1
1
u
u
0
3,1
2 3,2
2

0
0
u3,1
0
u3,2
= 0
(4.46)
u3,1 u3,2
0

Naviers equations then reduce to a single equation


3, + f3 = 0

(4.47)

2 u3 (x1 , x2 ) + f3 = 0

(4.48)

or
Now the boundary conditions follow from the Cauchys law in the following
form
u3
T3n = (u3 ) n =
(4.49)
n
where n denotes an outward unit normal on the boundary surface S.
Example 16 State the boundary conditions for antiplane strain model depicted
by Fig. ??.
80

s
0

r
s
0

l,m
2

l,m

a
2

Figure 4.3: An antiplane strain BVP.


Solution 15 For dierent surfaces the boundary conditions are stated as follows:
w
= T0
x
w
x = a, y h;
= T0
x
w
y = h; | x | a;
= T0
y
|

x |> a; 0 y h;

Plane State of Stress (PSS)


Definition 22 Plane state of stress parallel to xyplane has zz = zx =
zy 0.
Physically that corresponds to a thin plate in the xyplane loaded with
traction vectors in the same plane.
The strain field follows from
1
eij = [(1 + ) ij kk ij ]
E
81

to be
exx
eyy
ezz
exz
From

1
( xx yy )
E
1
=
( yy xx )
E

= (xx + yy )
E
= eyz = 0
=

(4.50)

ekk ij + eij ]
1 2
= 0 implies that
ij = 2[

we obtain first that zz

ezz =

(4.51)

(exx + eyy )
1

(4.52)

Based on this result the corresponding stress field is calculated to be


xx
yy
xy

2
E
(exx + eyy )
(exx + eyy ) =
1
1 2
E
=
(eyy + exx )
1 2
E
= 2exy =
exy
1+
=

(4.53)

where we have used the fact that E = 2(1 + ).


Note:
xx + yy

exx + eyy

E
(exx + eyy )
1
u v
+
x y

(4.54)

Equation of equilibrium becomes


xx xy
+
+ fx
x
y
xy
yy
+
+ fy
x
y

= 0
= 0

(4.55)

Substitution for stresses in terms of strains and then using eij = (ui,j +uj,i )/2
leads to the equations of equilibrium for PSS to be
1 + u v
2u 2u
+ 2)+
(
+
) + fx
x2
y
1 x x y
2v
1 + u v
2v
( 2 + 2 ) +
(
+
) + fy
x
y
1 y x y

82

= 0
= 0

(4.56)

or
2 u +

1+
u + f
1

= 0

u
u =
v

(4.57)

Plane Strain (PS) Model


Definition 23 We say that the model is of the plane strain type if
w 0; u = u(x, y); v = v(x, y)

(4.58)

Physically, this model can be envisioned in terms of a long cylinder along


the z-axis subjected to tractions which are uniform along the cylinders axis.
First, based on the prescribed from of the displacement field we obtain that
ezz = 0 implies that
zz = ( xx + yy )
(4.59)
Using that result the stresses (4.51) are calculated to be
xx
yy
xy

u v
(
+
)+
1 2 x y

u v
= 2[
(
+
)+
1 2 x y
u v
= (
+
)
y
x
= 2[

u
]
x
v
]
y
(4.60)

The Naviers equations for plane strain model become


2 u +

1
u + f
1 2

= 0

u
u =
v

(4.61)

By comparing Eqns. 4.61 and 4.57 it is evident that the only dierence
1+
1
between the two is in the factors 1
and 12
. Therefore, if in Eq. 3.35 is
replaced by / (1 + ), then Eqns. 4.61 and 4.57 will be identical. This suggests
that any solution of plane strain equation of equilibrium may be solved as a
plane stress problem by replacing the true value of by the apparent value
/(1 + ). Conversely, any plane stress problem may be solved as a plane strain
problem by replacing true by an apparent /(1 ).

83

4.4.3

Solution of 2D Elastostatic Problems in Terms of


Stresses

Equations of equilibrium
xx xy
+
+ fx
x
y
xy
yy
+
+ fy
x
y

= 0

(4.62)

= 0

(4.63)

consist of two equations with three unknowns. Thus we need to use the compatibility equation
2 exx
2 eyy
2 exy
+
=2
(4.64)
2
2
y
x
xy
By replacing the components of strain tensor in terms of stresses we obtain for
a plane strain model
1
[(1 2 ) xx (1 + ) yy ]
exx =
E
1
eyy =
[(1 2 ) yy (1 + ) xx ]
E
1+
exy =
(4.65)
xy
E
and consequently the compatibility equation becomes
2
2 xy
2
[(1

]
+
[(1

]
=
2
xx
yy
yy
xx
y 2
x2
xy

(4.66)

By adding Eqns. 4.62 and 4.63 we obtain


fx fy
2 xy
2 xx 2 yy
+
+
+
= 2
2
2
x
y
x
y
xy

(4.67)

Finally, by adding Eqns. 4.66 and 4.67 we obtain the compatibility equation for
a plane strain problem to be
1
fx
fy
(
+
)
1 x
y
2
2
+ 2
2
x
y

2 ( xx + yy ) =
2

(4.68)

Therefore a plane strain problem in terms of stresses is defined by partial


dierential equations
xx xy
+
+ fx
x
y
yy
xy
+
+ fy
x
y

= 0
= 0

2 ( xx + yy ) =
x

84

1
fx
fy
(
+
)
1 x
y

(4.69)

where D denote the interior of the body domain. The following boundary
conditions must be satisfied
Tx = xx nx + xy ny
Ty = yx nx + yy ny
x D

(4.70)

where D denotes the boundary of the domain.


Note for plane stress problem the compatibility equation becomes
2 ( xx + yy ) = (1 + )(

85

fx
fy
+
)
x
y

(4.71)

Chapter 5

Plane Elasticity in
Cylindrical Coordinates
5.1

Plane Strain (PS) Model

Let displacement filed in polar coordinates (r, ) for plane strain model be given
by
ur = ur (r, ); u = u (r, ); uz = 0
(5.1)
The corresponding strain and stress fields are given then (see Set #6)
1
1
(u, + ur ); ee = (ur, + ru,r u );
r
2r
ez = ezz = 0
2
d + 2ur,r ; = d +
(u, + ur ); zz = d;
r
ur
1
ur,r +
+ u,
r
r
1
u
z = 0; r = ( ur, + u,r )
r
r

err

= ur,r ; e =

erz

rr

d =
rz

(5.2)

(5.3)

Equations of equilibrium are then specified by (see Set#6)


1
1
rr,r + r, + (rr )fr
r
r
1
2
r,r + , + r + f
r
r

86

= 0
= 0

(5.4)

The strain-stress equations are given by


err

er

1
1+
( rr d) =
[(1 ) rr ]
2
E
1
1+
( d) =
[(1 ) rr ]
2
E
1+
r
E

(5.5)

Remark 12 Recall in Cartesian coordinates we have that


11
22
r + 22

= (e11 + e22 ) + 2e11


= (e11 + e22 ) + 2e22
= 2( + )(e11 + e22 ) = 2( + )d

while in polar coordinates it follows that


rr

rr +

= d + 2err
= d + 2e
= 2( + )(err + e ) = 2( + )d

Therefore
11 + 22 = rr +

(5.6)

Recall that the compatibility equation for PS model is


2 (11 + 22 ) +

1
f =0
1

which in polar coordinates becomes


(

5.2

1
1
1 2
fr
fr
1 f
2
+
+ 2 2 )( rr + ) +
(
+
+
)=0
2
r
r r r
1 r
r
r

(5.7)

Plane Stress (PSS) Model

In this case the model is defined by


rz = z = zz 0
rr

r
=
=
0
z
z
z
d = err + e + ezz
The stress-strain relations are defined by
rr

zz

= d + 2err
= d + 2e
= d + 2ezz 0
87

(5.8)

Vanishing of zz results in
ezz =

2
(err + e ) d =
(err + e )
+ 2
+ 2

Also
rr + = 2d + 2(err + e ) d =

rr +
3 + 2

(5.9)

(5.10)

Thus the strain-stress equations become


err

er

1
1
( rr d) = (rr )
2
E
1
( rr )
E
1+
r
E

(5.11)

Recall in Cartesian coordinates the compatibility equation for plane stress model
is given by
2 ( 11 + 22 ) + (1 + ) f =0
and
11
e33

= d + 2e11 ; 22 = d + 2e22 ;

2
=
(e11 + e22 ); d =
(e11 + e22 )
+ 2
+ 2

so that
11 + 22 = 2d + 2(e11 + e22 ) = (3 + 2)d
In polar coordinates zz = 0 implies that

2
(err + e ) d =
(err + e )
+ 2
+ 2
d = err + e + ezz

ezz

Consequently it follows that


rr + = 2d + 2(err + e ) = (3 + 2)d
Therefore, we have that
11 + 22 = rr +

(5.12)

and the compatibility equation becomes


(

1
fr
2
1 2
fr
1 f
+
+ 2 2 )( rr + ) + (1 + )(
+
+
)=0
2
r
r r r
r
r
r

88

(5.13)

5.3

Stress Function in Cartesian Coordinates

Assume that the body forces can be derived from a potential V so that
fx = V,x ; fy = V,y

(5.14)

Then, equilibrium equations become


( xx V ),x + xy,y
xy,x + ( yy V ),y

= 0
= 0

(5.15)

Let us introduce a function (x, y) such that


xx V
yy V
xy

= ,yy
= ,xx
= ,xy

(5.16)

Then the equations of equilibrium


,xyy ,xyy
,xxy + ,xxy

= 0
= 0

are trivially satisfied. The function is known as the Airys stress function.
Since the equilibrium equations are automatically satisfied by the stress function, we are left to check the compatibility equation (PSS)
2 ( xx + yy ) = (1 + )(fx,x + fy,y )
By substituting into the last equation for the stress function, the compatibility
equation reduces to
4 (x, y) = (1 )2 V
(5.17)
where
4 = ,xxxx + 2,xxyy + ,yyyy

(5.18)

For plane strain (PS) model the compatibility equation in terms of the stress
function becomes
1 2 2
4 (x, y) =
(5.19)
V
1

In the case of zero body forces, the compatibility equations (for both PSS
and PS) reduce to a single biharmonic equation
4 (x, y) = 0

89

(5.20)

5.3.1

Solution of a Biharmonic Equation

Lets consider a biharmonic equation


4 (x, y) = 2 (,xx + ,yy ) = 0

(5.21)

By introducing the mapping


z
z

= x + iy
= x iy

(5.22)

it follows that
2
4

2
2
2
+
=
4
x2 y 2
zz
4

= 2 2 = 16 2 2
z z
=

(5.23)
(5.24)

Therefore, the biharmonic equation reduces to


4 (z, z)
=0
z 2 z 2

(5.25)

Multiple integration of this equation results in


(z, z) = zF (z) + G(z) + zH(z) + J(z)

(5.26)

where F, G, H and J are arbitrary analytic functions of complex variable.


Remark 13 We have used the following convention for defining a conjugate
function G(z) of G(z)
G(z) = u(x, y) + iv(x, y)
G(z) = u(x, y) + i(x, y)
G(z) = u(x, y) iv(x, y)

(5.27)

Since we want to be real we require that


= 2 Im = 0
or
z([F (z) H(z) + z[(H(z) F (z)] + G(z) J(z) + J(z) G(z) = 0

(5.28)

Therefore, we must have that


F (z) = H(z) & G(z) = J(z)

(5.29)

and the stress function becomes


(z, z) = zF (z) + zF (z) + G(z) + G(z)
90

(5.30)

Let us express the analytic functions F and G as


F (z) =
G(z) =

1
[ (x, y) + i 2 (x, y)]
2 1
1
(x, y) + i 4 (x, y)
2 3

(5.31)

then the stress function reduces to


(x, y) = x 1 (x, y) + y 2 (x, y) + 3 (x, y)

(5.32)

where
1
2
3

= 2 Re F (z)
= 2 Im F (z)
= 2 Re G(z)

(5.33)

As we recall, F (z) and G(z) are assumed to be analytic functions.


Now for an analytic function f (z) = u(x, y) + iv(x, y) the real and imaginary
parts must satisfy Cauchy-Riemann equations
u,x = v,y

& u,y v,x

(5.34)

This implies that u and v must be harmonic functions in the region of analyticity
of f (z), i.e.,
2 u(x, y) = 0 & 2 v(x, y) = 0
(5.35)
Therefore we must have that
2 i = 0, i = 1, 2, 3

(5.36)

and Eq. 5.32 represents the most general solution of the biharmonic equation.
Remark 14 Often it is possible to avoid the follow this general procedure of
solving the biharmonic equation which is illustrated in the following two examples.
Consider stress function being a polynomial of the third degree
=

a3 3 b3 2
c3
d3
x + x y + xy 2 + y 3
6
2
2
6

(5.37)

This stress function automatically satisfies the biharmonic equation 4 = 0


and the stresses are calculated to be
xx c3 x + d3 y; yy = a3 x + b3 y; xy = b3 x c3 y

(5.38)

The constants a3,..., b3 can be chosen arbitrarily. However, stress function of the
form a polynomial of order four
=

c4
d4
e4
a4 4 b4 3
x + x y + x2 y 2 + xy 3 + y 4
12
6
2
6
12
91

(5.39)

in order to satisfy the biharmonic equation 4 = 0 the coecients must satisfy


the following
a4 + 2c4 + e4 = 0
(5.40)
Since 4 = 0 is a linear partial dierential equation the principle of superposition can be used in order to solve certain problems exactly.

5.3.2

Stress Function and Displacement Field

Plane Strain Case. For this model the strains can be calculated (see Set #8)
according to
1
exx =
[ xx ( xx + yy )]
2
Thus we have that
2u,x = ,yy 2

(5.41)

Similarly
eyy =

1
[yy ( xx + yy )]
2

leads to
2v,y = ,xx 2

(5.42)

Let us assume that we can find function such that


2 ,xy
Since

2 2 = 2 ,xy = 0

(5.43)
(5.44)

we obtain that must be a harmonic function, i.e.,


2 = 0

(5.45)

Equation 5.43 results in the following equations


,xx
,yy

= ,yy + ,xy
= ,xx ,xy

(5.46)
(5.47)

By substituting Eq. 5.46 into Eq. 5.41 and Eq. 5.47 into Eq. 5.42 we obtain
the following equations
2u,x
2v,y

= ,xx + (1 ) ,xy
= ,yy + (1 ) ,xy

(5.48)
(5.49)

Integration of the last two equation provides the following relationships between
the displacements and the stress function
2u = ,x + (1 ) ,y
2v = ,y + (1 ) ,x
92

(5.50)
(5.51)

2h

P
y
Figure 5.1: Bending of a cantiliver beam.
where
2 = ,xy

& 2 = 0

(5.52)

For plane stress (PSS) model corresponding equations are derived to be


2u = ,x + (1 + )1 ,y
2v

= ,y + (1 + )

,x

(5.53)
(5.54)

Example 17 Bending of a cantiliver beam loaded at its end section (Fig. ??.
Solution 16 Lets try
= b2 xy +

d4 3
xy
6

Then the stresses are calculated to be


xx

= ,yy = d4 xy

xy

= ,xy = b2

yy

= ,xx = 0

d4 2
y
2

Now the boundary conditions at y = h are


yy = 0

&

xy = 0 b2 =

d4 2
h
2

Solution 17 The force balance at the loaded end x = 0 can be stated as


Z h
Z h
d4
( xy )bdy = b
(b2 y 2 )dy
P =
2
h
h
d4
= b(b2 2h
2h3 )
23
93

which leads to the result

3P
2bh3
Consequently the stress filed can be determined to be
d4

xx =

3P xy
3P
; yy = 0; xy =
(h2 y 2 )
2bh3
4bh3

Using the moment of inertia I of the cross section I = 2bh3 /3 it follows then
that
P xy
P
xx =
(5.55)
; yy = 0; xy = (h2 y 2 )
I
2I
These results denote the exact solution provided the sharing forces at the end are
distributed according to the same parabolic law as the shearing stress xy in Eq.
5.55 and if the intensity of the normal forces at the built-in end is proportional
to y. If the forces at the ends are distributed in another way, the solution is not
exact. However, this solution is found to be acceptable for cross sections away
from the ends. As for the displacement field we have that
2u = ,x + ,y
2v = ,y + ,x

= 0 & 2 = ,xy

1 ; P S
=
(1 + )1 ; P SS

In order to find the function we proceed as follows. Equation


2 = 2 (b2 xy +

d4 3
xy ) = d4 xy = ,xy
6

implies that
d4 2 2
x y + f (x) + g(y)
4
Since 2 = 0 there follows that
=

d4 2
(x + y 2 ) = 0
2
d4
d4
f 00 (x) + x2 = g 00 (y) y 2 = const = a0
2
2

f 00 (x) + g 00 (y) +

So

d4
d4 2 2 a0 2
x y + (x y 2 ) (x4 + y 4 ) + a1 x + b1 y + c1
4
2
24
Based on this result we can evaluate
d4
d4
,x = b2 y + y 3 ; ,y = b2 x + xy 2
6
2
d4 2
d4 3
,x =
xy + a0 x x + a1
2
6
d4 2
d4 3
,y =
x y a0 y y + b1
2
6
=

94

so that the displacement filed can be evaluated to be


2u = ,x + ,y
d4
d4
d4
= b2 y + y 3 + ( x2 y a0 y y 3 + b1 )
6
2
6
2v = ,y + ,x
d4
d4
d4
= b2 x + xy 2 + ( xy 2 + a0 x x3 + a1 )
2
2
6

(5.56)

or
(1 + )d4 3
d4 2
x y
y (a0 b2 )y + b1
2
6
(1 )d4 2 d4 3
=
xy
x + (a0 + b2 )x + a1
2
6

2u =
2v

These solutions cannot satisfy the required boundary conditions at x =


u = v = 0; x = ; h < y < h
However the unknown constants a0 , a1 , b1 can be determined by using an approximate engineering boundary conditions at x =
u = v = 0; v,x = 0; x = ; y = 0
Thus
u( , 0) = b1 = 0 b1 = 0
d4 2
d4
+ (a0 + b2 ) = 0 a0 + b2 =
v,x ( , 0) =
2
2
d4 3 d4 3
d4 3
v( , 0) =
+
+ a1 = 0 a1 =
6
2
3

(5.57)

Therefore the displacement filed is calculated to be (for PSS = (1 + )1 )


2v(x, 0) =

d4 3
x + (a0 + b2 )x + a1
6

or

P
(x3 3 2 x + 2 l3 )
6EI
The strength of material solution is given by
v(x, 0) =

v(0, 0) =

P 3
3EI

It should be emphasized that our solution is an approximate one since it does not
satisfy all the boundary conditions. The results become more and more accurate
as the distance from the ends increases.

95

5.4

Airy Stress Function in Polar Coordinates

Recall for plane strain model in polar coordinates we have the strain-displacement
equations of the form
= ur,r
1
=
(u, + ur )
r
1
=
(ur, + ru,r u )
2r
= ez = ezz = 0

err
e
er
erz

(5.58)

Corresponding equilibrium equations are


1
1
rr,r + r, + ( rr ) + fr
r
r
1
2
r,r + , + r + f
r
r
Stress-strain equations are specified by
err

= 0
= 0

1+
[(1 ) rr ]
E
1+
[(1 ) rr ]
E

(5.59)

(5.60)

or
= (1 ) rr = + (1 )(rr + )
= (1 ) rr = rr + (1 )( rr + )

2err
2e

(5.61)

Equilibrium equations for zero body force can be written as


(r rr ),r + r,
(r2 r ),r + r ,

= 0
= 0

(5.62)

In order to introduce the stress function in polar coordinates we define first the
transformation from Cartesian to polar coordinates
x = r cos ; y = r sin
y
x

r2

= x2 + y 2 ; = tan1

r,x

= cos ; r,y = sin ; ,x =

sin
cos
; ,y =
r
r

(5.63)

Therefore, we have that


,y
,yy
,xx

= ,r r,y + , ,y = ,r sin + ,
1
= ( ,r +
r
1
= ( ,r +
r

cos
r

1
1
) cos2 + ,rr sin2 + ( , ),r sin 2
r2 ,
r
1
1
2
2
) sin + ,rr cos ( , ),r sin 2
r2 ,
r
96

(5.64)
(5.65)
(5.66)

s
q

q r

s
xy

r
q

rr

xx

y
s

rq

xy

rq

rr

yy

q r

s
q

Figure 5.2: Two elements in equilibrium.


Lets consider now two elements of a solid as depicted by Fig.5.2. Equilibrium
conditions for the first element yield
xx = sin2 + rr cos2 r sin 2 = ,yy

(5.67)

while the equilibrium of the second provides


yy = cos2 + rr sin2 + r sin 2 = ,xx

(5.68)

Then from Eqns. 5.65 to 5.68 it follows that


rr

r

1
1
+
r ,r r2 ,
= ,rr
1
= ( , ),r
r
=

(5.69)

which expresses stresses in terms of the stress function in polar coordinates.


It is easy to verify that stresses calculated from Eqns. 5.69 identically satisfy
equations of equilibrium Eq. 5.62.
Now the compatibility equation in cylindrical coordinates is given by
2 ( xx + yy ) = 2 ( rr + ) = 0
or
4 = (

2
1
2
1
1 2
1 2
+
+ 2 2 )( 2 +
+ 2 2) = 0
2
r
r r r r
r r r

(5.70)

Also,
2 = ,xx + ,yy = xx + yy = rr +
From strain-stress equations 5.60 it follows that
2ur,r = + (1 )(rr + )
97

(5.71)

or
2ur,r = ,rr + (1 )2 ;

(P S)

(5.72)

Now lets introduce an auxiliary function such that


2 = (r , ),r

(5.73)

then Eqn. 5.72 becomes


2ur,r = ,rr + (1 )(r , ),r
which after integration results in
2ur = ,r + 1 )r , ;

(P S)

(5.74)

Similarly, Eqns.5.60,5.69 and 5.73 result in


1
2 (ur + u, ) = rr + (1 )( rr + )
r
1
1
= ( ,r + 2 , ) + 1 )2
r
r
From the last result it follows then that
2u,

1
= ,r , + (1 )r2 + ,r (1 )r ,
r
1
= , + (1 )r[2 , ]
r
1
= , + (1 )r2 ,r
r

which after integration provides


1
2u = , + (1 )r2 ,r ;
r

(P S)

(5.75)

Therefore, Eqns. 5.74 and 5.75 relate stress function and auxiliary function
to displacement field in polar coordinates. In order to examine further the
properties of the auxiliary function we recall that
er =

1
r
1 1
(ur, + ru,r u ) =
= ( , ),r
2r
2
2 r

Therefore

1
2(ur, + ru,r u ) = 2r( , ),r
r

or
r + (1 )r , + r(
1
+ , (1 )r2 ,r
r

2
2,r
r ,
98

1
1
+ 2(1 )r ,r + (1 )r2 ,rr )
r2 , r ,r

or

1
1
(1 )r3 ( ,rr + ,r + 2 , ) = 0
r
r
Thus we have that the auxiliary function must satisfy
2 = 0

(5.76)

By replacing with /(1 + ) we obtain the corresponding plane stress


equations relating the displacement field to stress function and the auxiliary
function according to
1
r
1 + ,
1
1 2
= , +
r ,r
r
1+

2ur

= ,r +

2u

5.4.1

(5.77)
(5.78)

Solutions for Stress Function in Polar Coordinates

Lets consider a special case of stress function of the form


(r, ) = gn (r) cos n

(5.79)

Then we have that


2
1
1 2
+
+ 2 2 )gn (r) cos n
2
r
r r r
d2 gn 1 dgn n2
= ( 2 +
2 gn ) cos n = Gn (r) cos n
dr
r dr
r

2 = (

(5.80)

and thus
d2
1 d
n2 2
+
) gn (r) cos n

dr2
r dr
r2
2
2
1 d
n
d
2 )Gn (r) cos n
= ( 2+
dr
r dr
r

4 = (

(5.81)

Thus biharmonic equation 4 = 0 becomes


(

1 d
1 d
d2
n2 d2
n2
+
)(
+
)gn (r) = 0

dr2
r dr
r2 dr2
r dr
r2

(5.82)

Case n=0
In this case Eqn. 5.82 becomes
(

1 d d2
1 d
d2
+
+
)(
)g0 (r) = 0
dr2
r dr dr2
r dr

Let
G0 (r) = (

d2
1 d
+
)g0
dr2
r dr
99

(5.83)

(5.84)

Then Eqn. 5.83 becomes


r2 G000 + rG00 = 0;

G0 =

dG
dr

(5.85)

Now introduction of new variable t via


r = et

dt
1
=
dr
r

(5.86)

results in
dG
dr
d2 G
dr2

=
=

1 dG
r dt
d dG
1 d 1 dG
1 dG
1 d2 G
(
)=
(
)= 2
+ 2 2
dr dr
r dt r dt
r dt
r dt

Consequently Eqn. 5.85 becomes


d2 G0
=0
dt2

(5.87)

G0 (t) = A + Bt

(5.88)

which can be solved for


Now from Eqn. 5.84 it follows that
(

d2
1 d
1
d
1 d2 g0
d2
+
)g0 = 2 (r2 2 + r )g0 = 2 2 = A + Bt
2
dr
r dr
r
dr
dr
r dt

(5.89)

Integrating the last result twice produces


g0 (t) =

1
B
(A B)e2t + te2t + Ct + D
4
4

(5.90)

or
g0 (r) = a0 r2 + b0 r2 ln r + c0 + d0 ln r = (r, ); n = 0

(5.91)

Case n=1
In this case Eqn. 5.82 becomes
d2
1 d
1
+
)2 g1 = 0
dr2
r dr r2

(5.92)

d2
1 d
1
+
)g1 (r) = G1 (r)
dr2
r dr r2

(5.93)

r2 G001 + rG01 G1 = 0

(5.94)

(
Let
(
then Eqn. (5.92) implies

Using the substitution defined by Eqn. 5.86, Eqn. 5.94 becomes


d2 G1
G1 = 0
dt2
100

(5.95)

which solution is given by


G1 (t) = Aet + Bet

(h9e39)

Therefore Eqn. 5.93 can be written in the form


(

1 d
d2
1
+
2 )g1
2
dr
r dr r

=
=

or

1 2 d2
d
(r
+ r 1)g1
2
2
r
dr
dr
1 d2 g1
(
g1 ) = Aet + Bet
r2 dt2

d2 g1
g1 = Ae3t + Bet
(5.96)
dt2
The homogeneous and particular solutions of the last equation are given by
g1h
g1p

= b1 et + d1 et
= a1 e3t + c1 tet

(5.97)
(5.98)

and the solution is specified by


g1 (t) = a1 e3t + b1 et + c1 tet + d1 et
or
g1 (r) = a1 r3 + b1 r + c1 r ln r + d1 r1

(5.99)

Case n>1
In this case Eqn. 5.82 becomes
(

1 d
d2
n2
+
2 )2 gn (r) = 0
2
dr
r dr
r

(5.100)

As before, let
Gn (r) = (

d2
1 d
n2
+
)gn (r)

dr2
r dr
r2

(5.101)

Then Eqn. 5.100 implies


1
n2
G00n + G0n 2 Gn = 0
r
r

(5.102)

Introducing a change of variable according to Eqn. 5.86 the last equation becomes
d2 Gn
n2 Gn = 0
(5.103)
dt2
which can be solved to give
Gn = Aent + Bent

101

(5.104)

Consequently
(

d2
1 d
n2
+
)gn (r) =

dr2
r dr
r2
=

1 2 d2
d
(r
+ r n2 )gn
2
2
r
dr
dr
1 d2
2
(
n )gn = Gn
r2 dt2

which implies that


(

d2
n2 )gn (t) = Ae(n+2)t + Be(n+2)t
dt2

(5.105)

Homogeneous and particular solutions of the last equation are given by


gnh
gnp

= bn ent + dn ent
= an e(n+2)t + cn e(n+2)t

(5.106)
(5.107)

So the solution for gn is given by


gn (t) = an e(n+2)t + bn ent + cn e(n+2)t + dn ent
or
gn (r) = an rn+2 + bn rn + cn rn+2 + dn rn ; n > 1

5.4.2

(5.108)

Stress and Displacement Fields

Based on the stress function just derived we proceed with evaluation of corresponding stress and displacement fields. For that purpose we utilize the following equations

rr

r

1
1
+
r ,r r2 ,
= ,rr
1
= ( , ),r
r
=

2ur

= ,r + r ,
1
2u = , + r2 ,r
r
(1 ); P S
=

(1 + )1 ; P SS
2 = (r , ),r

= 0

Case n=0
102

= a0 r2 + b0 r2 ln r + c0 + d0 ln r
1
1
d0
rr =
+ = 2a0 + b0 (1 + 2 ln r) + 2
r ,r r2 ,
r
d0
= ,rr = 2a0 + b0 (3 + 2 ln r) 2
r
r = 0

(5.109)
(5.110)
(5.111)

For displacement field we have


2ur = ,r + r ,

(5.112)

Now since
2 = (rr + ) = (r , ),r
4a0 + 4b0 (1 + ln r) = (r , ),r
we get that
c + 4a0 r + 4b0 r ln r = r ,

(5.113)

Using the last results in Eqn. 5.112 results in


2ur = (2a0 r + 2b0 r ln r + b0 r +

d0
) + (4a0 r + 4b0 r ln r + c)
r

or
2ur = 2(2 1)a0 r + b0 r + 2(2 1)b0 r ln r

d0
+ 2u0r
r

Now from Eqn. 5.113 we get that


c
c
, = 4a0 + 4b0 ln r + = 4a0 + 4b0 ln r + + D
r
r
and thus
r2 ,r = 4b0 r

(5.114)

(5.115)

Based on this result we get the tangential component of displacement


1
2u = , + r2 ,r + 2u0
r
or
2u = 4b0 r + 2u0

(5.116)

= (a1 r3 + b1 r + c1 r ln r + d1 r1 ) cos
c1 2d1
rr = (2a1 r +
3 ) cos
r
r
c1 2d1
= (6a1 r +
+ 3 ) cos
r
r
c1 2d1
r = (2a1 r +
3 ) sin
r
r

(5.117)

Case n=1

103

(5.118)
(5.119)
(5.120)

Since
2c1
) cos = (r , ),r
r
= (4a1 r2 + 2c1 ln r) cos + c

2 = rr + = (8a1 r +
r ,

the displacement field ur follows from Eqn. 5.74 to be


2ur

= ,r + r ,
= [(3 4)a1 r2 + b1 + (1 2)c1 ln r + c1

d1
] cos
r2

+2u0r

(5.121)

Based on the result that


,

r2 ,r

2c1
c
ln r) cos +
r
r
2c1
c
= (4a1 r +
ln r) sin + + D
r
r
= (4a1 r2 2c1 ln r + 2c1 ) sin c
= (4a1 r +

it follows from Eqn. 5.75 that


2u

= [(1 + 4)a1 r2 + b1 + 2c1 + (1 2)c1 ln r +

d1
] sin
r2

+2u0

(5.122)

Case n>1
In this case the stress function is given by
= (an rn+2 + bn rn + cn rn+2 + dn rn ) cos n
and the corresponding stress fields is given by
rr

r

= [(n 2)(n + 1)an rn + (n 1)nbn rn2


+(n 1)(n 2)cn rn + n(n + 1)dn rn2 ] cos n
= [(n + 2)(n + 1)an rn + n(n 1)nbn rn2
+(n 2)(n 1)cn rn + n(n + 1)dn rn2 ] cos n
= n[an (n + 1)rn + bn (n 1)rn2 + cn (1 n)rn
(n + 1)dn rn2 ] sin

Since
2 = rr + = (r , ),r
we have that
r , = (4an rn+1 + 4cn rn+1 ) cos n

104

(5.123)
(5.124)
(5.125)
(5.126)

Therefore, Eqn. 5.74 produces


2ur

= [(n + 2 4Ga)rn+1 + nbn rn1 (n 2 + 4)cn rn+1


ndn rn1 ] cos n + 2u0r
(5.127)

Similarly, from the result


,
r2 ,r

= 4(an rn + cn rn ) cos n

= 4(an rn+1 cn rn1 ) sin n

we obtain from Eqn. 5.75 the tangential component of displacement to be


2u

5.4.3

= [(n + 4)an rn+1 + nbn rn1 + (n 4)rn+1 cn


+ndn rn1 ] sin n + 2u0

(5.128)

Axisymmetric Case

Plane Strain Model and n=0


= 1
2ur = 2(1 2)a0 r b0 r + 2(1 2)b0 r ln r d0 r1
+2u0r
or
ur

1+
[2(1 2)a0 r b0 r + 2(1 2)b0 r ln r d0 r1 ] + u0r
E
4(1 2 )
b0 r + u0
E

Plane Stress Case


= (1 + )1
1
ur =
[2(1 )a0 r (1 )b0 r + 2(1 )b0 r ln r
E
(1 + )d0 r1 ] + u0r
4b0 r
u =
+ u0
E
Example 18 Consider a semicircular arch of inner and outer radii being a and
b, respectively loaded as shown by Fig. 5.3
Determine the stress and displacement fields.

105

2P

r
Q
M

P
d

Q
M

Figure 5.3: A semicircular arch problem.


Solution 18 The boundary conditions along the curved surfaces of the arch are
specified by
(5.129)
rr = r = 0; r = a, b; 0 < <
while along the end section = 0, we must have
Z

dr

= P ; = 0,

r dr

= Q; = 0,

a
b

rdr

= M P d

(5.130)

Lets try the stress function of the form


= f (r) + g (r)(A cos + B sin )

(5.131)

Then from Eqns. 5.91 and 5.99 we get


f (r) = a0 r2 + b0 r2 ln r + c0 + d0 ln r
g(r) = a1 r3 + b1 r + c1 r ln r + d1 r1

106

(5.132)

The stress filed follows from Eqns. 5.109 to 5.111 and 5.118 to 5.120 to be
rr

r

= 2a0 + b0 (1 + 2 ln r) + d0 r2 + (2a1 r + c1 r1 2d1 r3 )


(A cos + B sin )
= 2a0 + b0 (3 + 2 ln r) d0 r2 + (6a1 r + c1 r1 + 2d1 r3 )
(A cos + B sin )
= (2a1 r + c1 r1 2d1 r3 )(A sin B cos )
(5.133)

Then the boundary conditions (5.129) become


2a0 + b0 (1 + 2 ln a) + d0 a2
2a0 + b0 (1 + 2 ln b) + d0 b2
2a1 a + c1 a1 2d1 a3
2a1 b + c1 b1 2d1 b3

=
=
=
=

0
0
0
0

(5.134)

The end conditions (5.130) then become


B[a1 r2 + c1 ln r + d1 r2 ]ba
A[3a1 r2 + c1 ln r d1 r2 ]ba
[a0 r2 + b0 r2 (1 + ln r) d0 ln r]ba

= Q
= P
= (M + P d)

The unknown coecients are found to be


M0 2
[b a2 + 2(b2 ln b a2 ln a)]
N2
2M0 2
(b a2 )
N2
4M0 2 2 b
a b ln
N2
a

a0

b0

d0

b
= (M + P d); N2 = (b2 a2 )2 4a2 b2 (ln )2
a
A = P ; B = Q
a2 b2
b
; N1 = a2 b2 + (a2 + b2 ) ln
d1 =
2N1
a
1
a2 + b2
a1 =
; c1 =
2N1
N1

M0

107

(5.135)

P
O

y
r
q

x
Figure 5.4: A normal line load on an elastic half-space.
Then the stress field is given by
rr

4M0 a2 b2 b
r
r
( 2 ln b2 ln + a2 ln )
N2
r
a
b
a
a2 b2 a2 + b2
P cos Q sin
(r + 3
)
+
N1
r
r
4M0 a2 b2 b
r
r
=
(
ln b2 ln + a2 ln + a2 b2 )
N2 r2
a
b
a
a2 b2 a2 + b2
P cos Q sin
(3r 3
+
)
N1
r
r
P sin Q cos
a2 b2 a2 + b2
=
(r + 3
)
N1
r
r
=

(5.136)

If M = Q = 0 we get
rr

P cos
a2 + b2 a2 b2
(r
+ 3 )
N1
r
r
2
2
P cos
a +b
a2 b2
(3r
3 )
N1
r
r
P sin
a2 + b2
a2 b2
(r
+ 3 )
N1
r
r

(5.137)

Problem 1 Normal Line Load on a Half-Space


Lets consider the problem depicted by Fig.(5.4).
We try now to solve this problem by choosing the following stress function
=

P
r sin

108

(5.138)

y
-s

rr

x
Figure 5.5: Local stress field on a small semicircle at the origin.
Then the stress field becomes
rr

r

1
1
2P
+ =
cos
r ,r r2 ,
r
= ,rr = 0
1
= ( , ),r = 0
r
=

(5.139)

It should be noted that the surface of the half-space is stress free (excluding
the origin). For that reason it is necessery to examine the situation under the
load more carefuly. Suppose we cut-out an infinitesimal semicircle of material
at the origin as shown by Fig(5.5).
Due to the original loading, the resultant force acting on the cylinder C must
be in the x-direction and of magnitude P. Then
Z

/2

2P
rr r cos d =

/2

/2

cos2 d = P

/2

independently of as 0. From the stress-strain relationship


(1 + )
[(1 ) rr ]
E
1
(1 + )
(u, + ur ) =
[(1 ) rr ]
r
E
1 1
u
r
( ur, + u,r ) =
2 r
r
2

err

= ur,r =

er

109

(5.140)

we get
ur,r

2(1 2 )P
cos
Er
2(1 + )P
cos
Er

1
(u, + ur ) =
r
u
1 1
( ur, + u,r ) = 0
2 r
r

(5.141)
(5.142)
(5.143)

The from Eqn.(5.141) we have that


ur =

2(1 2 )P
ln r cos + g 0 ()
E

(5.144)

and then from Eqn.(5.142) follows


u =

2(1 + )P
2(1 2 )P
sin +
ln r sin g() + f (r)
E
E

(5.145)

for an arbitrary functions f and g. By substituting (5.144) and (5.145) into


(5.143) we obtain
g 00 () + g () +

2(1 + )(1 2)P


sin = rf 0 (r) + f (r) = const = 0 (5.146)
E

where primes denote dierentiation. The last equation can be solved for
g
f

(1 + )(1 2)P
cos + a cos + b sin
E
= dr

(5.147)
(5.148)

for arrbitrary constants a, b, and c.


Since Eqn.(5.148) represents the rigid-body motion (RBM) we may assume
d = 0. Consequently, the displacement fields can be calculated from Eqns.(5.144)
and (5.145)
ur
u

2(1 2 )P
(1 + )(1 2)P
ln r cos
sin + u0r (5.149)
E
E
2(1 + )
2(1 2 )P
=
P sin +
ln r sin
E
E
(1 + )(1 2)P
(5.150)
(sin cos ) + u0
+
E
=

where the superscript 0 denotes the RBM.


In order to determine the RBMs we assume: i) all the points on the x-axis
do not have any lateral displacement, and ii) one point at distance d from the
origin along the x-axis is fixed. Thus
ur |=0,r=d = 0 u0r =
110

2(1 2 )P
ln d cos
E

which imples
ur

(1 + )
d
P [2(1 ) cos ln (1 2) sin ]
E
r
(1 + )
d
P [sin 22(1 ) sin ln (1 2) cos ]
E
r

(5.151)
(5.152)

where we have used for convenience u0 = 2(1 ) ln d sin . We should note


that u ( = 0) = 0.
At the half-space surface ( = /2) we have thus
(1 + )(1 2)P
2E
(1 + )P
d
u =
[1 2(1 + ) ln ]
(5.153)
E
r
which imples that the material is displacemed toward the origin.
We should note that the solution is not valid at the point of the load application. Also, there is a log-singularity at infinity.
For a PSS model, the stress and displacement fileds can be obtained by
replacing with /(1+), while keeping = E/{2(1+)} unchanged. Therefore
we have that the displacement field is given by
ur

P
d
[2 cos ln (1 ) sin ]
E
r
P
d
u =
[(1 + ) sin 2 sin ln (1 ) cos ]
E
r
Problem 2 Stress in a wedge subjected to a line loade at the vertex.
ur

(5.154)

Consider an infinite elastic wedge subjected to a load P per unit length at


the vertex along the axis of the wedge (Fig.(5.6)).
Letss try the stress function
= cr sin

(5.155)

which implies the following stress field (PS)


1
1
2c
+ =
cos
r ,r r2 ,
r
= ,rr = 0
1
r = ( , ),r = 0
r
2c
zz = ( rr + ) =
cos
r
Lets consider the equilibrium of the OAB portion of the wedge.
Z
rr r cos d + P = 0

Z
rr r sin d = 0
rr

111

(5.156)

(5.157)
(5.158)

y
B
P
O
2a

rr

A
Figure 5.6: The wedge problem.
By substittuting the value of rr in the last two equations we get from
Eqn.(5.157)
P
c=
2 + sin 2
and thus
2P cos
rr =
(5.159)
; = r = 0
r(2 + sin 2)
Note that
2P
rr ( = /2) =
cos
r
which is the same result as for the half-space problem.
Problem 3 Wedge with a line load perpendicular to the x-axis.
Lets consider the second wedge problem depicted by Fig.(5.141).
In this case we try
= dr cos

(5.160)

which provides
rr

2d
sin
r
= r = 0
=

Balance of forces on the portion of the wedge can be stated as


Z
Q
rr r sin d + Q = 0 d =
2

sin 2

112

(5.161)
(5.162)

y
Q
q

O
2a

rr

Figure 5.7: Second wedge problem.


and thus
rr

5.4.4

2Q
sin
r(2 sin 2)
= r = 0
=

(5.163)
(5.164)

Stresses Near a Corner

Consider an elastic wedge as shown by Fig.(5.8).


We recall that the stress and displacement fields can be evaluated in terms
of the Stress function and the auxiliary function , where
4 = 0 & 2 = 0 & 2 = (r , ),r

(5.165)

= r+1 F () & = rm G()

(5.166)

Lets try
where and m are unknown constants while F and G are unknown functions.
By substituting Eqn.(5.166) into (5.165) we get
(

1
2
1 2
+
+ 2 2 )2 r+1 F () = 0
2
r
r r r

or
[( 1)2 +

d2 2
] F () = 0
d2

113

O
2q

r
x

Figure 5.8: An elastic wedge.


The last equation becomes
F (iv) + [( + 1)2 + ( 1)2 ]F 00 + ( 2 1)2 F = 0

(5.167)

Similarly for the function we get


G00 + m2 G = 0

(5.168)

Characteristic equation for Eqn.(5.167) is given by


4 + [( + 1)2 + ( 1)2 ]2 + ( 2 1)2 = 0
with the corresponding roots
2I = ( 1)2 ;

2II = ( + 1)2

(5.169)

Therefore, the solution for the two functions can be written as


1)
F () = b1 sin( + 1) + b2 cos + 1) + b3 sin( 1) + b4 cos( (5.170)
G() = a1 cos m + a2 sin m
(5.171)
Using the third equation of Eqn.(5.165) we must have
[( + 1)2 F () + F 00 ()]r1 = (m + 1)rm G0 ()

(5.172)

In order for Eqn.(5.172) to be vaild for 0 < r < , we must have


1=m
114

(5.173)

Then from Eqns.(5.170) ,(5.171) and (5.172) it follows that


a1 =

4b3
;
1

a2 =

4b4
1

(5.174)

Based on these results functions and become

= r+1 [b1 sin( + 1) + b2 cos( + 1) + b3 sin( 1) + b4 cos( (5.175)


1)
4
(5.176)
=
[b3 cos( 1) + b4 sin( 1)]r1
1
Now displacement filed can be evaluates according to
2ur
2u

= ,r + r ,
1
= , + r2 ,r
r

which provides
2ur
2u

= {( + 1)[b1 sin( + 1) + b2 cos( + 1)]


+(4 1)[b3 sin( 1) + b4 cos( 1)]}r
= {( + 1)[b1 cos( + 1) b2 sin( + 1)]
(4 + 1)[b3 cos( 1) b4 sin( 1)]}r

(5.177)
(5.178)

Now the stress field can be calculated using Eqns.(5.156) and (5.175)
rr

r

= {( + 1)[b1 sin( + 1) + b2 cos( + 1)]


+( 3)[b3 sin( 1) + b4 cos( 1)]}r1
= ( + 1)[b1 sin( + 1) + b2 cos( + 1)]
+b3 sin( 1) + b4 cos( 1)]r1
= {( + 1)[b1 cos( + 1) b2 sin( + 1)]
+( 1)[b3 cos( 1) b4 sin( 1)]}r1

(5.179)
(5.180)
(5.181)

Since the both wedge faces are stress free we have that
= r = 0;

= 0

(5.182)

Using Eqns.(5.180)-(5.182) we obtain


b1 sin( + 1)0 + b2 cos( + 1)0 + b3 sin( 1)0 + b4 cos( 1)0
b1 sin( + 1)0 + b2 cos( + 1)0 b3 sin( 1)0 + b4 cos( 1)0
( + 1)[b1 cos( + 1)0 b2 sin( + 1)0 ] + ( 1)[b3 cos( 1)0 b4 sin( 1)0 ]
( + 1)[b1 cos( + 1)0 + b2 sin( + 1)0 ] + ( 1)[b3 cos( 1)0 + b4 sin( 1)0 ]
or
b1 sin( + 1)0 + b3 sin( 1)0
b1 ( + 1) cos( + 1)0 + b3 ( 1) cos( 1)0
b2 cos( + 1)0 + b4 cos( 1)0
b2 ( + 1) sin( + 1)0 + b4 ( 1) sin( 1)0
115

=
=
=
=

0
0
0
0

(5.183)
(5.184)
(5.185)
(5.186)

=
=
=
=

0
0
0
0

uq
y

uq

ur

ur

x
-q

x
-q

antisymmetric

symmetric

ur

ur

uq

uq

Figure 5.9: Symmetric and antisymmetric wedge displacements.


For a nontrivial solution of the systems (5.183)-(5.184) and (5.185)-(5.186)
we must have
( 1) sin( + 1)0 cos( 1)0 ( + 1) cos( + 1)0 sin( 1)0
( 1) sin(11)0 cos( + 1)0 ( + 1) cos( 1)0 sin( + 1)0

= 0
= 0

or
sin 20 sin 20
sin 20 + sin 20

= 0
= 0

(5.187)
(5.188)

Therefore, for a wedge with both faces free, must satisfy Eqn.(5.187) and
it produces antisymmetric displacements (see the b1 and b3 components of
displacements in Eqns.(5.177) and (5.178)). In order to produce symmetric
displacements (b2 and b4 terms) must satisfy Eqn.(5.188).The symmetric
and antisymmetric diplacements for the wedge are shown by Fig.(5.9).
The solutions for are in general complex numbers. For the displacement
field to be finite everywhere we need to have (see Eqns.(5.177) and (5.178))
Re > 0

(5.189)

For the faces of the wedge being clamped, the boundary conditions are
ur = u = 0;

116

= 0

(5.190)

Stressfree Wedge
2.5

2
=0.9

symmetric

1.5

0.5

0.5

1.5
antisymmetric

2.5
2

1.5

0.5

0.5

1.5

Figure 5.10: Equations for for a stress-free wedge with 0 = 0.9.


Then from Eqns.(5.190),(5.177), and (5.178) we obtain
b1 ( + 1) sin( + 1)0 b3 (4 1) sin( 1)0
b1 ( + 1) cos( + 1)0 + b3 (4 + 1) cos( 1)0
b2 ( + 1) cos( + 1)0 b4 (4 1) cos( 1)0
b2 ( + 1) sin( + 1)0 + b4 (4 + 1) sin( 1)0

=
=
=
=

0
0
0
0

(5.191)
(5.192)
(5.193)
(5.194)

Thus for a nontrivial solution for the two systems (5.191),(5.192) and (5.193),(5.194)
we have
(4 1) sin 20 + sin 20
(4 1) sin 20 sin 20

= 0
= 0

(5.195)
(5.196)

where Eqns.(5.195) and (5.196) refer to antisymmatric and symmetric solution,


respectively.
It is apparent from Eqns.(5.179)-(5.181) that the stresses are proportional
to r1 . Thus, the stresses are finite at the corner for Re 1 and unbounded
for Re < 1.
Lets evaluate the Eqns.(5.187) and (5.188). We choose the angle 0 = 0.9
and /3. The results are displayed by Figs.(5.10) and (5.11).
117

Stressfree Wedge
3

= /3
0

antisymmetric
1

2
symmetric

3
2

1.5

0.5

0.5

1.5

Figure 5.11: Equations for for a stress-free wedge and 0 = /3.

118

r








Figure 5.12:
It can be shown that for 0 < 0 < /2 the stresses are finite at the corner
(e.g., Fig.(5.11)). For /2 < 0 < , the stresses are unbounded at the corner
(e.g., Fig.(5.10)). For the details see Williams[?].
Problem 4 Circular Hole in a Plate
Lets consider a problem of a plate as shown by Fig.(5.12).
5.199Therefore, in the far field we must have
xx = 0 ;

yy = xy = 0

(5.197)

Thef from the equilibrium of the elements depicted by Fig.(??) we have that
rr

r

= xx cos2 + yy sin2 + xy sin 2


= xx sin2 + yy cos2 xy sin 2
= (yy xx ) sin cos + xy cos 2

(5.198)

Consequently, Eqns.(5.197) and (5.198) produce


rr

r

0
(1 + cos 2)
2
0
=
(1 cos 2)
2
0
= sin 2
2
=

(5.199)

Lets try the stress function of the form


= f (r) + g(r) cos 2

119

(5.200)

Then from the general results in polar coordinates we have that


f (r) = A ln r + Br2 ln r + Cr2 + D
g(r) = r2 + r4 + r2 +

(5.201)
(5.202)

Since the stresses must be finite when r , we get B = = 0.


From general results for stresses in polar corrdinates (with n = 0 and n = 2)
by taking into account that the stresses should remain bounded in the far filed,
we have that
rr
rr
r

A
6
4
+ 2C (2 + 4 + 2 ) cos 2
r2
r
r
A
6
= 2 + 2C + (2 + 4 ) cos 2
r
r
6
2
= (2 4 2 ) sin 2
r
r
=

(5.203)

Now the boundary conditions at the hole are given by


rr = 0;

r = 0;

r=a

(5.204)

which become
A
6
4
+ 2C (2 + 4 + 2 ) cos 2
a2
a
a
6
2
(2 4 2 ) sin 2
a
a

= 0
= 0

Since at infinity the fields repesented by Eqns.(5.199) and (5.203) must be


the same we have that
0
2C 2 cos 2 =
(1 + cos 2)
2
0
2C + 2 cos 2 =
(1 cos 2)
2
0
2 sin 2 = sin 2
2
and thus

0
0
; =
4
4
so that the boundary conditions at the hole imply
C=

A = 2Ca2 =
6
4
+ 2
4
a
a
6
2
2 4 2
a
a

2 +

Consequently, we obtain
0
A = a2 ;
2

0 2
a
2

= 0
= 0

0 2
a ;
2

120

0 4
a
4

and the stress filed becomes


rr

r

0
3a4 4a2
a2
[(1 2 ) + (1 + 4 2 ) cos 2]
2
r
r
r
0
3a4
a2
=
[(1 + 2 ) (1 + 4 ) cos 2]
2
r
r
0
3a4 2a2
= (1 4 + 2 ) sin 2]
2
r
r
=

The maximum stress is then given by


max |r=a = 3 0 ;

r = a; = /2, 3/2

121

(5.205)

Chapter 6

Torsion
6.1

Saint-Venant Theory

The paper by Saint-Venant (1853) to the French Academy contains not only the
authors theory of torsion but it also gives an account of all that was known at
that time in the theory of elasticity.
Lets consider an elastic cylinder with an axis z, and with the ends at z = 0
and z = L (Fig.(6.1)).
The shaft is subjected at its ends to a ditributed shearing stresses whose
resultant moment is a torque T. Lets define the rotation angle about the zaxis
as . Then recall that
d
T
=
(6.1)
dz
J
where J is the polar moment of inertia and is the shear modulus.
Lets define the cross section od the shaft perpedicular to the zaxis as
shown by Fig.(6.2).
For components of displacement field ui , Saint-Venant assumed that as the
shaft twistas the plane ross-sections are worped but the projections on the
xyplane rotate as a rigid body, i.e.,
ur
u
uz

= 0
= rz
= (x, y)

(6.2)

Since
ur
u

= ux cos + uy sin
= ux sin + uy cos

we have that
ux
uy

= ur cos u sin
= ur sin + u cos
122

(6.3)
(6.4)

x
T
z
Figure 6.1: An elastic cylinder.

y

r

zy

zx


R

O
/R=C

n
Figure 6.2: Cross-section of the cylinder subjected to torsional loading.

123

Then from Eqns.(6.2)-(6.4) we have


u = yz
v = xz
w = (x, y)

(6.5)

where we have used notation u = ux , v = uy , and w = uz .


The equaions of equilibrium ij,j = 0 become
xx,x + xy,y + xz,z
yx,x + yy,y + yz,z
zx,x + zy,y + zz,z

= 0
= 0
= 0

(6.6)

while the boundary condtions on the lateral surface C are


xx nx + xy ny
yx nx + yy ny
zx nx + zy ny

= 0
= 0
= 0

(6.7)

Boundary conditions at z = 0 and z = L can be written as (see Fig.(6.2))


zz

= 0

(6.8)

zx dxdy

= 0

(6.9)

zy dxdy

= 0

(6.10)

(xzy y zx )dxdy

= T

(6.11)

Z
Z

ZR

where T is the torque applied at the ends. Equations (6.9)-(6.11) state that the
stress field zx and zy are equipollent to a torque T.
Now Eqns.(6.2) imply that the following stresses are zero
xy = xx = yy = zz = 0

(6.12)

while the noziro components of the stress tensor are


xz
yz

y)
x

= (
+ x)
y
= (

(6.13)
(6.14)

Therefore, the equations of equilibrium (6.6) reduce to


2 2
+ 2 = 0;
x2
y
124

(x, y) R

(6.15)

Boundary conditions (6.7) become

(6.16)
= ynx xny ; on C
n
The boundary conditions (6.9)-(6.11) now imply
Z
Z
zx dxdy =
(,x y)dxdy
R
R
Z n

x(,x y) ,x + x(,y + x) ,y dxdy


=
ZR

x(,x y)nx + x(,y + x)ny dx


=

Zc

=
x
ynx + xny ds = 0
n
C

due to Eqn.(6.16). Similarly, it follows that


Z
zy dxdy = 0
R

so it follows that the end conditions (6.9) and (6.10) are automatically staified.
The end condition (6.11) becomes
T = J
where
J=

(6.17)

(x2 + y 2 + x,y y,x )dxdy

the polar moment of inertia when the cross-section is circular. The same symbol
is retained for noncircular cross-section.
Therefore, the problem of torsion reduces to solving the following boundary
value problem
2 (x, y) = 0; (x, y) R
(6.18)

(6.19)
= ynx xny ; (x, y) C
n
The ends sections of the shaft are free to warp, and if stresses prescribed on
the end sections are exactly the same as those given by the solution, then the
exact solution is obtained, and the solution is unique. If the strerss distribution
acting on the end sections, while equipollent to the tprque T, does not agree
exactly with those given by Eqns.(6.13) and (6.14), then only an approximate
solution is obtained.
According to the principle proposed by Saint-Venant, the rror in the approximation is signicant only in the neighborhood of the end sections.
According to the Divergence Theorem, we have that
Z
Z

2 dxdy =
ds = 0
(6.20)
R
C n
Therefore, condition for existence of the a solution is that the integral of
/n calculated over the entire boundary C must vanish.
125

6.2

Prandtl Theory

In this approach we take the stress components as the principal unknowns. If


we assume that xz and yz are nonzero, then the only equilibroum equation
to be cosidered is
zx,x + zy,y = 0
(6.21)
Prandtl proposed to use a stress function (x, y) so that
xz = ,y ;

yz = ,x

(6.22)

Evidently, the equation of equiliberium (6.21) is automatically satisfied. The


boundary condtions on C and the end conditions are specified by Eqns.(6.7)(6.11). The compatibility conditons are given by
eij,lk + ekl,ij eik,jl ejl,ik = 0

(6.23)

Using the inverse of the Hookes law


eij =

1+

ij kk ij
E
E

the compatibility conditions become

(,kl ij + ,ij kl ,jl ik (6.24)


,ik jl
1+
= kk

ij,lk + kl,ij ik,jl jl,ik

Since only six of the 81 equations are linearly independent, by setting k = l


in the last equation and by taking into account that ij,j = 0 (no body forces)
the compatibilty equations reduce to
ij,kk +

,ij
,kk ij = 0
1+
1+

Since kk = 0, we get that compatibilty equations become


2 xz
2 yz

= 0
= 0

which i terms of the stress function implies

) = 0
y

2 (
) = 0
x
2 (

2
=0
y
2
=0
x

or
2 (x, y) = const in R
On the boundary C we have that boundary conditions
zx nx + zy ny = 0
126

(6.25)

become

dy dx
d
+
=
=0
y ds
x ds
ds

or
(x, y) = const on C
For a simply connected regions without loss of generality we can set
(x, y) = 0 on C

(6.26)

We still need to examine the end conditions. The condition zz = 0 is


satisfied by the starting assumptions. As for the other conditions we have
Z
Z
Z

zx dxdy =
ny ds = 0
dxdy =
R
R y
C
where we have used Eqn.(6.26). Similarly
Z
Z
Z

zy dxdy =
nx ds = 0
dxdy =
R
R x
C
and
T

(x zy yzx )dxdy
Z
Z
= (x ,x + y y )dxdy = [(x),x + (y),y 2]dxdy
Z R
ZR
dxdy
= (xnx + yny )ds + 2
R

which implies
T =2

dxdy

Therefore, in Prandtls formulation the torsion of a shaft can be posed as


following boundary-value problem
2 (x, y) = const; f or (x, y) R
= 0; f or (x, y) C
Z
2
dxdy = T

(6.27)
(6.28)
(6.29)

We still need to determine the constant in Eqn.(5.183). This can be done


through boundary conditions on displacements. Namely, from
zx
zy

= (w,x + u,z ) = (w,x y)


= (w,y + v,z ) = (w,y + x)

we have that
xz,y yz,x = 2
127

or
2 (x, y) = 2

(6.30)

where we have used Eqns.(6.22). By comparing Eqns.(6.30) and (6.27) we see


that the unknwn constant is equal to -2.
Therefore, Prandtls torsion problem becomes
2 (x, y) = 2; f or (x, y) R
= 0; f or (x, y) C
Z
2
dxdy = T

(6.31)
(6.32)
(6.33)

xz

= ,y ;

yz = ,x

(6.34)

Example 19 Torsion of an elliptic bar.


In that case the cross-section boundary C is defined by
C:

x2 y 2
+ 2 =1
a2
b

(6.35)

Lets try the Prandtls stress function of the form


= A(

x2 y 2
+ 2 1)
a2
b

(6.36)

Then, using Eqn.(6.31) we obtain


A=
and therefore
=

a2 b2

+ b2

a2

a2 b2
x2 y 2
(
+ 2 1)
a2 + b2
a2
b

(6.37)

Now Eqn.(6.33) becomes


T =

2a2 b2

a2 + b2

x2 y 2
+ 2 1)dxdy
a2
b

Consequently, we cosider the integral


Z
x2 y 2
( 2 + 2 1)dxdy
I =
b
R a
Z 2 2
Z
a

1x /a

dx

= 2

4b
=
3

(1

x2
y2
+
1)dy
a2
b2

x2 3/2
ab
) dx =
2
a
2

128

(6.38)

Based on the last result qn.(6.38) states that


T =

a3 b3

a2 + b2

(6.39)

The stress field is calculated to be


2T y
ab3
2T x
=
a3 b

zx

= ,y =

(6.40)

zy

= ,x

(6.41)

The resultant stress can be calculated according to


resul

= ( 2xz + 2yz )1/2


2T 1
x2 1
1
[ 2 2 ( 2 2 )]1/2
ab b
a b
a

The maximum resulting stress is then


max =

2T
;
ab2

at x = 0; y = b

Saint-Venant Approach
Since the stresses zx = ,y and zy = ,x are known, we have that
xz
yz

= (,x y) = ,y
= (,y + x) = ,x

(6.42)
(6.43)

Equation (6.42) provides that


,x = y +
and thus
=

1
b2 a2
y
,y = 2

a + b2

b2 a2
xy + f (y)
a2 + b2

(6.44)

Using Eqns.(6.43) we have that


,y = x

1
b2 a2
x
,x = 2

a + b2

and using Eqn.(6.44) we get


f 0 (y) = const = 0
which implies that the worping function is given by
=

a2 b2
xy
a2 + b2
129

(6.45)

C
+ +

Figure 6.3: w=const lines for an elliptical shaft under torsional loading.
Consequently, the displacement component w is given by
w = =

a2 b2
xy
a2 + b2

(6.46)

Therefore, the curves w = const are represented by a family of hyperbolas as


shown by Fig.(6.3).
It is evident from Eqn.(6.46) that for a corcular shaft (a = b) there is no
worping of the cross-sections z = const, i.e., w = 0.

130

You might also like